You are on page 1of 46

WB JEE

Engineering Entrance Exam

Solved Paper 2019


Physics
Category I (Q. Nos. 1 to 30) Assuming that a proton is 2000 times heavier
than an electron, what will be the relation
Carry 1 marks each and only one option is
between the de Broglie wavelength of the
correct. In case of incorrect answer or any
proton (λ p) and that of electron (λ e)?
combination of more than one answer, 1/4 mark
λe
will be deducted. (a) λ p = 2000λe (b) λ p =
2000
λ
1. A ray of light is reflected by a plane mirror. (c) λ p = 20 5 λe (d) λ p = e
e$ 0 , e$ and n$ be the unit vectors along the 20 5
incident ray, reflected ray and the normal to 4. To which of the following the angular
the reflecting surface respectively. velocity of the electron in the n-th Bohr orbit
Which of the following gives an expression is proportional?
for e$ ? 1 1 1
(a) n2 (b) (c) (d)
n2 n3 / 2 n3

n 5. In the circuit shown, what will be the current
∧ ∧ through the 6V zener?
e0 e
1 kΩ
A

(a) e$ 0 + 2(e$ 0 ⋅ n$ )n$ (b) e$ 0 − 2(e$ 0 ⋅ n$ )n$ 10 V 1 kΩ 6V


(c) e$ 0 − (e$ 0 ⋅ n$ )n$ (d) e$ 0 + (e$ 0 ⋅ n$ )n$

2. A parent nucleus X undergoes α-decay with a B


half-life of 75000 yrs. The daughter nucleus (a) 6 mA, from A to B (b) 2 mA, from A to B
Y undergoes β-decay with a half-life of 9 (c) 2 mA, from B to A (d) Zero
months. In a particular sample, it is found
that the rate of emission of β-particles is 6. Each of the two inputs A and B can assume
nearly constant (over several months) at values either 0 or 1. Then which of the
10 7 /h. What will be the number of α-particles following will be equal to A ⋅ B?
emitted in an hour? (a) A + B (b) A + B
(a) 10 2
(b) 10 7
(c) 10 12
(d) 10 14 (c) A ⋅ B (d) A + B

3. A proton and an electron initially at rest are 7. The correct dimensional formula for impulse
accelerated by the same potential difference. is given by

Page 1 of 46
(a) ML2 T− 2 (b) MLT− 1 (c) ML2 T− 1 (d) MLT− 2 13. A compressive force is applied to a uniform
8. The density of the material of a cube can be rod of rectangular cross-section so that its
estimated by measuring its mass and the length decreases by 1%. If the Poisson’s ratio
length of one of its sides. If the maximum for the material of the rod be 0.2, which of
error in the measurement of mass and length the following statements is correct?
are 0.3% and 0.2% respectively, the “The volume approximately ...........”
maximum error in the estimation of the (a) decreases by 1% (b) decreases by 0.8%
density of the cube is approximately. (c) decreases by 0.6% (d) increases by 0.2%
(a) 1.1% (b) 0.5% (c) 0.9% (d) 0.7% 14. A small spherical body of radius r and density
ρ moves with the terminal velocity v in a fluid
9. Two weights of the mass m1 and m2(> m1) are
of coefficient of viscosity η and density σ.
joined by an inextensible string of negligible What will be the net force on the body?
mass passing over a fixed frictionless pulley. 4π 3
The magnitude of the acceleration of the (a) r (ρ − σ ) g (b) 6πηrv
3
loads is (c) Zero (d) Infinity
m2 − m1
(a) g (b) g
m2 15. Two black bodies A and B have equal surface
m1 m − m1 areas and are maintained at temperatures
(c) g (d) 2 g
m2 + m1 m2 + m1 27ºC and 177ºC respectively. What will be the
ratio of the thermal energy radiated per
10. A body starts from rest, under the action of second by A to that by B?
an engine working at a constant power and (a) 4 : 9 (b) 2 : 3 (c) 16 : 81 (d) 27 : 177
moves along a straight line. The displacement
s is given as a function of time ()
t as 16. What will be the molar specific heat at
(a) s = at + bt 2 , a and b are constants constant volume of an ideal gas consisting of
rigid diatomic molecules?
(b) s = bt , b is a constant
2
3 5
(c) s = at 3 / 2 , a is a constant (a) R (b) R
2 2
(d) s = at , a is a constant (c) R (d) 3R
11. Two particles are simultaneously projected in 17. Consider the given diagram. An ideal gas is
the horizontal direction from a point P at a contained in a chamber (left) of volume V
certain height. The initial velocities of the and is at an absolute temperature T. It is
particles are oppositely directed to each other allowed to rush freely into the right chamber
and have magnitude v each. The separation of volume V which is initially vacuum. The
between the particles at a time when their whole system is thermally isolated. What will
position vectors (drawn from the point P) are be the final temperature of the equilibrium
mutually perpendicular, is has been attained?
v2 v2
(a) (b)
2g g
4v 2 2v2
(c) (d) V V
g g Ideal gas Vacuum

12. Assume that the earth moves around the sun


in a circular orbit of radius R and there exists
a planet which also move around the sun in a T
circular orbit with an angular speed twice as (a) T (b)
2
large as that of the earth. The radius of the T
orbit of the planet is (c) 2T (d)
4
R
(a) 2 − 2 / 3 R (b) 2 2 / 3 R (c) 2 − 1/ 3 R (d)
2

Page 2 of 46
18. Five identical capacitors, of capacitance 20µF 22. An electric current ‘I’ enters and leaves a
each, are connected to a battery of 150 V, in a uniform circular wire of radius r through
combination as shown in the diagram. What diametrically opposite points. A particle
is the total amount of charge stored? carrying a charge q moves along the axis of
the circular wire with speed v. What is the
magnetic force experienced by the particle
when it passes through the centre of the circle?
µ 0i µ 0i
(a) qv (b) qv
a 2a
µ i
(c) qv 0 (d) Zero
2 πa

(a) 15 × 10− 3 C 23. A current ‘I’ is flowing along an infinite,


(b) 12 × 10− 3 C straight wire, in the positive Z-direction and
the same current is flowing along a similar
(c) 10 × 10− 3 C
parallel wire 5 m apart, in the negative
(d) 3 × 10− 3 C Z-direction. A point P is at a perpendicular
distance 3m from the first wire and 4m from
19. Eleven equal point charges, all of them
the second. What will be magnitude of the
having a charge + Q, are placed at all the
magnetic field B of P?
hour positions of a circular clock of radius r,
5 7
except at the 10 h position. What is the (a) (µ 0 I) (b) (µ 0 I)
electric field strength at the centre of the 12 24
5 25
clock? (c) (µ 0 I) (d) (µ 0 I)
Q 24 288
(a) from the centre towards the mark 10
4 πε0 r 2 24. A square conducting loop is
Q placed near an infinitely long
(b) from the mark 10 towards the centre
4 πε0 r 2 current carrying wire with one
(c)
Q
from the centre towards the mark 6
edge parallel to the wire as shown
4 πε0 r 2 in the figure. If the current in the
(d) Zero. straight wire is suddenly halved,
which of the following statements I
20. A negative charge is placed at the midpoint will be true?
between two fixed equal positive charges, “The loop will .............”.
separated by a distance 2d. If the negative (a) stay stationary
charge is given a small displacement x (x < < d) (b) move towards the wire
perpendicular to the line joining the positive (c) move away from the wire
charges, how the force (F) developed on it will (d) move parallel to the wire
approximately depend on x?
1 25. What is the current I shown in the given
(a) F ∝ x (b) F ∝ circuit?
x
1 R R R
(c) F ∝ x 2
(d) F ∝
x2

21. To which of the following quantities, the 2R V 2R 2R 2R 2R I


radius of the circular path of a charged
particle moving at right angles to a uniform
magnetic field is directly proportional?
(a) energy of the particle V V
(a) (b)
(b) magnetic field 2R R
(c) charge of the particle V V
(c) (d)
(d) momentum of the particle 16R 8R

Page 3 of 46
26. When the value of R in the balanced 30. When the frequency of the light used is
Wheatstone bridge, shown in the figure, is changed from 4 × 10 14 s− 1 to 5 × 10 14 s− 1 , the
increased from 5Ω to 7 Ω, the value of S has to angular width of the principal (central)
be increased by 3Ω in order to maintain the maximum in a single slit Fraunhoffer
balance. What is the initial values of S? diffraction pattern changes by 0.6 radian.
What is the width of the slit (assume that
P Q the experiment is performed in vacuum)?
. × 10− 7 m
(a) 15 (b) 3 × 10− 7 m
G −7
(c) 5 × 10 m (d) 6 × 10− 7 m
R S
Category II (Q. Nos. 31 to 35)
Carry 2 marks each and only one option is
correct. In case of incorrect answer or any
(a) 2.5 Ω (b) 3 Ω combination of more than one answer, 1/2 mark
(c) 5 Ω (d) 7.5 Ω will be deducted.
27. When a 60 mH inductor and a resistor are 31. A capacitor of capacitance C is connected in
connected in series with an AC voltage source, series with a resistance R and DC source of
the voltage leads the current by 60º. If the emf E through a key. The capacitor starts
inductor is replaced by a 0 .5 µF capacitor, the charging when the key is closed. By the
voltage lags behind the current by 30º.What is time the capacitor has been fully charged,
the frequency of the AC supply? what amount of energy is dissipated in the
1 1
(a) × 104 Hz (b) × 104 Hz resistance R?
2π π
3 1
(c) × 104 Hz (d) × 108 Hz C
2π 2π
R
28. A point object is placed on the axis of a thin E
convex lens of focal length 0.05 m at a distance
of 0.2 m from the lens and its image is formed 1 E2
(a) CE 2 (b) 0 (c) CE 2 (d)
on the axis. If the object is now made to 2 R
oscillate along the axis with a small amplitude
of A cm, then what is the amplitude of 32. A horizontal fire hose with a nozzle of
5
oscillation of the image? cross-sectional area × 10 − 3 m 2 delivers
 1  21
you may assume, 1 + x ≈ 1 − x , where x < < 1 a cubic metre of water in 10s. What will be
  the maximum possible increase in the
4A 5A temperature of water while it hits a rigid
(a) × 10− 2 m (b) × 10− 2 m
9 9 wall (neglecting the effect of gravity)?
A A
(c) × 10− 2 m (d) × 10− 2 m (a) 1ºC (b) 0.1ºC (c) 10ºC (d) 0.01ºC
3 9
33. Two identical blocks of ice move in opposite
29. In Young’s experiment for the interference of directions with equal speed and collide with
light, the separation between the slits is d and each other. What will be the minimum
the distance of the screen from the slits is D. If speed required to make both the blocks
D is increased by 0.5% and d is decreased by melt completely, if the initial temperatures
0.3% then for the light of a given wavelength, of the blocks were − 8 ºC each?
which one of the following is true? (Specific heat of ice is 2100 Jkg − 1 K − 1 and
“The fringe width ............. latent heat of fusion of ice is 3.36 × 105 Jkg − 1 )
(a) increases by 0.8% (b) decreases by 0.8% (a) 840 ms − 1 (b) 420 ms − 1
−1
(c) increases by 0.2% (d) decreases by 0.2% (c) 8.4 ms (d) 84 ms − 1

Page 4 of 46
34. A particle with charge q moves with a velocity 37. The initial pressure and volume of a given
v in a direction perpendicular to the  Cp 
directions of uniform electric and magnetic mass of an ideal gas  with = γ  , taken in
fields, E and B respectively, which are  CV 
mutually perpendicular to each other. Which a cylinder fitted with a piston, are p 0 and V0
one of the following gives the condition for respectively. At this stage the gas has the
which the particle moves undeflected in its same temperature as that of the surrounding
original trajectory? medium which is T0 . It is adiabatically
V
y compressed to a volume equal to 0 .
2
E
Subsequently the gas is allowed to come to
thermal equilibrium with the surroundings.
v
x What is the heat released to the
B surrounding?
p0 V0
(a) 0 (b) (2 γ − 1 − 1)
z γ −1
E B E B p0 V0
(a) v = (b) v = (c) v = (d) v = q (c) γp0 V0 ln2 (d)
B E B E 2(γ − 1)

35. A parallel plate capacitor in series with a 38. A projectile thrown with an initial velocity of
resistance of 100 Ω, an inductor of 20 mH 10 ms − 1 at an angle α with the horizontal,
and an AC voltage source of variable has a range of 5 m. Taking g = 10 ms − 2 and
frequency shows resonance at a frequency of neglecting air resistance, what will be the
1250 estimated value of α?
Hz.
π (a) 15º (b) 30º (c) 45º (d) 75º
If this capacitor is charged by a DC voltage 39. In the circuit shown in the figure all the
source to a voltage 25 V, what amount of resistance are identical and each has the
charge will be stored in each plate of the value r Ω. The equivalent resistance of the
capacitor? combination between the points A and B will
(a) 0.2 µC (b) 2 mC (c) 0.2 mC (d) 0.2 C remain unchanged even when the following
pairs of points marked in the figure are
Category III (Q. Nos. 36 to 40) connected through a resistance R.
Carry 2 marks each and one or more option(s) 1 2 3 4 5
is/are correct. If all correct answers are not marked r r r r r r
and also no incorrect answer is marked then score A B
= 2 × number of correct answers marked ÷ actual r 6 r 7 r
number of correct answers. If any wrong option is
marked or if any combination including a wrong (a) 2 and 6 (b) 3 and 6
option is marked, the answer will be considered (c) 4 and 7 (d) 4 and 6
wrong, but there is no negative marking for the 40. A metallic loop is placed in a uniform
same and zero marks will be awarded. magnetic field B with the plane of the loop
perpendicular to B. Under which condition(s)
36. Electrons are emitted with kinetic energy T given an emf will be induced in the loop?
from a metal plate by an irradiation of light
of intensity J and frequency ν. Then, which “If the loop is ............”
of the following will be true? (a) moved along the direction of B
(b) squeezed to a smaller area
(a) T ∝ J
(c) rotated about its axis
(b) T linearly increasing with ν
(d) rotated about one of its diameters
(c) T ∝ time of irradiation
(d) Number of electrons emitted ∝ J

Page 5 of 46
Chemistry
Category I (Q. Nos. 41 to 70) 44. The compound, which evolves carbon dioxide
Carry 1 mark each and only one option is correct. on treatment with aqueous solution of
In case of incorrect answer or any combination of sodium bicarbonate at 25ºC, is
more than one answer, 1/4 mark will be (a) C 6H5OH (b) CH3COCI
deducted. (c) CH3CONH2 (d) CH3COOC 2H5

45. The indicated atom is not a nucleophilic site


41. One of the products of the following reaction
in
is P.
(a) BH−4 (b) CH3MgI
O ↑ ↑
(c) CH3 OH (d) CH3 NH2
(i) aq. KOH ↑ ↑
CCl3
(ii) H3O+
P 46. The charge carried by 1 millimole of M n +
ions is 193 coulombs. The value of n is
Structure of P is (a) 1 (b) 2 (c) 3 (d) 4
O O 47. Which of the following mixtures will have
CO2H H
the lowest pH at 298 K?
(a) (b) (a) 10 mL 0.05 N CH3COOH + 5 mL 0.1 N NH4OH
(b) 5 mL 0.2 N NH4Cl + 5 mL 0.2 N NH4OH
O HO (c) 5 mL 0.1N CH 3COOH +10 mL 0.05 N CH 3COONa
O
Cl (d) 5 mL 0.1 N CH3COOH + 5 mL 0.1 N NaOH
OH
(c) (d)
Cl 48. Consider the following two first order
reactions occurring at 298 K with same initial
42. For the reaction below, the product is Q. concentration of A :
(1) A → B; rate constant, k = 0 .693 min − 1
CO2H
Acetic anhydride
Q [C9H8O4] (2) A → C ; half-life, t1 / 2 = 0 .693 min
Conc.H2SO4(cat.)heat Choose the correct option.
HO
(a) Reaction (1) is faster than reaction (2).
The compound Q is (b) Reaction (1) is slower than reaction (2).
CO2H COOCOCH3 (c) Both reactions proceed at the same rate.
(d) Since two different products are formed, rates
(a) (b) cannot be compared.

OCOCH3 OH
49. For the equilibrium, H 2O( l ) - H 2O(v),
which of the following is correct?
CO2H CO2H (a) ∆G = 0, ∆H < 0, ∆S < 0
COCH3 (b) ∆G < 0, ∆H > 0, ∆S > 0
(c) (d) (c) ∆G > 0, ∆H = 0, ∆S > 0
COCH3 (d) ∆G = 0, ∆H > 0, ∆S > 0
 ab 
OH OH 50. For a van der Waals’ gas, the term  
represents some  V2
43. Cyclopentanol on reaction with NaH followed
(a) pressure (b) energy
by CS2 and CH 3I produces a/an
(c) critical density (d) molar mass
(a) ketone (b) alkene
(c) ether (d) xanthate 51. In the equilibrium, H 2 + I 2 -
2HI, if at a
given temperature the concentrations of the

Page 6 of 46
reactants are increased, the value of the 60. The melting points of (i) BeCl 2 (ii) CaCl 2 and
equilibrium constant, K C , will (iii) HgCl 2 follows the order
(a) increase (a) i < ii < iii (b) iii < i < ii
(b) decrease (c) i < iii < ii (d) ii < i < iii
(c) remain the same
61. Which of these species will have non-zero
(d) cannot be predicted with certainty
magnetic moment?
52. If electrolysis of aqueous CuSO 4 solution is (a) Na + (b) Mg (c) F − (d) Ar +
carried out using Cu-electrodes, the reaction
taking place at the anode is 62. The first electron affinity of C, N and O will
− be of the order
(a) H + e → H
+

(b) Cu2 + (aq ) + 2e − → Cu (s ) (a) C < N < O (b) N < C < O


(c) C < O < N (d) O < N < C
(c) SO 24 − (aq ) − 2e − → SO 4
(d) Cu(s ) − 2e − → Cu2 + (aq ) 63. The H N H angle in ammonia is 107.6º,
while the H P H angle in phosphine is
53. Which one of the following electronic 93.5º. Relative to phosphine, the p-character
arrangements is absurd? of the lone-pair on ammonia is expected
(a) n = 3, l = 1, m = − 1 (b) n = 3, l = 0, m = 0 to be
(c) n = 2, l = 0, m = − 1 (d) n = 2, l = 1, m = 0 (a) Less (b) More
(c) Same (d) Cannot be predicted
54. The quantity hv / K B corresponds to
64. The reactive species in chlorine bleach is
(a) wavelength (b) velocity
(a) Cl 2O (b) OCl −
(c) temperature (d) angular momentum
(c) ClO 2 (d) HCl
55. In the crystalline solid MSO 4 ⋅ nH 2O of molar 65. The conductivity measurement of a
mass 250 g mol − 1 , the percentage of coordination compound of cobalt (III) shows
anhydrous salt is 64 by weight. that it dissociates into 3 ions in solution. The
The value of n is compound is
(a) 2 (b) 3 (c) 5 (d) 7
(a) hexaamminecobalt(III) chloride
56. At S.T.P. the volume of 7.5 g of a gas is 5.6 L. (b) pentaamminesulphatocobalt(III) chloride
The gas is (c) pentaamminechloridocobalt(III) sulphate
(d) pentaamminechloridocobalt(III) chloride
(a) NO (b) N2O
(c) CO (d) CO 2 66. In the Bayer’s process, the leaching of
125
alumina is done by using
57. The half-life period of 53 I
is 60 days. The (a) Na 2CO 3 (b) NaOH (c) SiO 2 (d) CaO
radioactivity after 180 days will be
67. Which atomic species cannot be used as a
(a) 25% (b) 12.5% nuclear fuel?
(c) 33.3% (d) 3.0% 233 235 239 238
(a) 92 U (b) 92 U (c) 94 Pu (d) 92 U
58. Consider, the radioactive disintegration
68. The molecule/molecules that has/have
82 A
210
→ B→ C → 82 D
206
delocalised lone pair(s) of electrons is/are
The sequence of emission can be O O
(a) β, β, β (b) α, α ,β N
(a) OCH3 (b) C
(c) β, β, γ (d) β , β , α H 3C CH2 H 3C CH2
59. The second ionisation energy of the following OCH3
elements follows the order (c) (d) CH3CH==CHCH2NHCH3
(a) Zn > Cd < Hg (b) Zn > Cd > Hg
(c) Cd > Hg < Zn (d) Zn < Cd < Hg
(a) I, II and III (b) I, II and IV
(c) I and III (d) only III

Page 7 of 46
69. The conformations of n-butane, commonly 73. At constant pressure, the heat of formation
known as eclipsed, gauche and of a compound is not dependent on
anti-conformations can be interconverted by temperature, when
(a) rotation around C  H bond of a methyl group (a) ∆C p = 0 (b) ∆C v = 0
(b) rotation around C  H bond of a methylene (c) ∆C p > 0 (d) ∆C p < 0
group
(c) rotation around C1-C2 linkage 74. A copper coin was electroplated with Zn and
(d) rotation around C2-C3 linkage then heated at high temperature until there
is a change in colour. What will be the
70. The correct order of the addition reaction resulting colour?
rates of halogen acids with ethylene is (a) white (b) black
(a) hydrogen chloride > hydrogen bromide (c) silver (d) golden
> hydrogen iodide
(b) hydrogen iodide > hydrogen bromide
75. Oxidation of allyl alcohol with a peracid gives
> hydrogen chloride
a compound of molecular formula C3H 6O 2 ,
which contains an asymmetric carbon atom.
(c) hydrogen bromide > hydrogen chloride
The structure of the compound is
> hydrogen iodide
(d) hydrogen iodide > hydrogen chloride OH
> hydrogen bromide O
(a) O OH (b) CH3
H
Category II (Q. Nos. 71 to 75)
H H
Carry 2 marks each and only one option is O
correct. In case of incorrect answer or any OH
(c) H3C (d) H3C
combination of more than one answer, 1/2 mark O OH
will be deducted.
71. The total number of isomeric linear Category III (Q.Nos. 76 to 80)
dipeptides which can be synthesised from Carry 2 marks each and one or more option(s)
racemic alanine is is/are correct. If all correct answers are not
(a) 1 (b) 2 (c) 3 (d) 4 marked and also no incorrect answer is marked
72. The kinetic study of a reaction like vA → P at then score = 2 × number of correct answers
marked ÷ actual number of correct answers. If
300 K provides the following curve, where
−3 any wrong option is marked or if any combination
concentration is taken in mol dm and time
including a wrong option is marked, the answer
in min.
will considered wrong, but there is no negative
r0 : Initial rate
[A0] : Initial
marking for the same and zero mark will be
concentration of A awarded.
√r0
Slope = 4.0 76. Haloform reaction with I2 and KOH will be
responded by
O
O
[A]0 Me
(a) I Ph (b) Ph
Identify the correct order(n) and rate constant(k) OH
(a) n = 0, k = 4.0 mol dm− 3 min− 1 OH O
(b) n = 1 / 2, k = 2.0 mol1/ 2 dm− 3 / 2 min− 1
(c) n = 1, k = 8.0 min− 1 (c) Me
Me
Ph (d) Ph N Me
H
(d) n = 2, k = 16.0 dm3 mol − 1 min− 1

Page 8 of 46
77. Identify the correct statement(s): (a) NH4NO 3 (b) NaCl
(a) The oxidation number of Cr in CrO 5 is + 6. (c) (NH4 )2 CO 3 (d) HCOONa
(b) ∆H > ∆U for the reaction N2O 4 (g ) → 2NO 2 (g ), 80. The compound(s), capable of producing
provided both gases behave ideally. achiral compound on heating at 100ºC
(c) pH of 01 . N H2SO 4 is less than that of 0.1 N HCl is/are
at 25ºC. Me Me
(d) 
RT  Et CH2COOH Et CH2CO2H
 = 0.0591 volt at 25ºC.
 F  (a) (b)
78. Compounds with spin-only magnetic moment HO2C CO2H HO2C CO2H
equivalent to five unpaired electrons are Me

(a) K 4 [Mn(CN)6 ] (b) [Fe(H2O)6 ] Cl 3 Me H


Et CO2H Et CO2H
(c) K 3 [FeF6 ] (d) K 4 [MnF6 ] (d)
(c)
79. Which of the following chemicals may be used O Me O Me
to identify three unlabelled beakers containing
conc.NaOH, conc.H 2SO 4 and water?

Mathematics
Category I (Q. Nos. 1 to 50) 4. The value of the integration
π /4
Carry 1 mark each and only one option is  µ sin x 
correct. In case of incorrect answer or any ∫  λ |sin x | +
 1 + cos x
+ γ  dx

− π /4
combination of more than one answer, 1/4 mark
will be deducted. (a) is independent of λ only
(b) is independent of µ only
1. lim (x n ln x), n > 0 (c) is independent of γ only
x → 0+ (d) depends on λ, µ and γ

1  sin 2 t 
(a) does not exist (b) exists and is zero a a
sin 2 t
(c) exists and is 1 (d) exists and is e − 1 5. The value of lim ∫ e dt − ∫e dt is
x→0x
 y x + y 
 x
2. If ∫ cos x log  tan  dx equal to
 2 2 2
y
 x (a) esin (b) e 2sin y (c) e| sin y | (d) ecosec y

= sin x log  tan  + f (x), then f (x) is equal to


 2 x x
6. If ∫ 22 ⋅ 2x dx = A ⋅ 22 + C , then A is equal to
(assuming c is a arbitrary real constant)
1 1
(a) c (b) c − x (c) c + x (d) 2 x + c (a) (b) log2 (c) (log 2 )2 (d)
log 2 (log 2 )2
 1− x
3. y = ∫ cos 2 tan − 1  dx is an equation of 7. The value of the integral
 1+ x
1
 x 2015 1 
a family of ∫  e| x |(x 2 + cos x) + e| x |  dx is equal to
(a) straight lines (b) circles − 1 
(c) ellipses (d) parabolas (a) 0 (b) 1 − e −1
(c) 2e − 1 (d) 2(1 − e − 1 )

Page 9 of 46
3  n n n 1  1 
8. lim 1 + + + 14. If log 62 + = log 2 2 x + 8  , then the values
n→ ∞ n
 n+3 n+6 n+9 2x  
 
n  of x are
+ ... + 
n + 3(n − 1)  1 1
(a) ,
1 1
(b) , (c) −
1 1
,
1
(d) , −
1
4 3 4 2 4 2 3 2
(a) does not exist (b) is 1
(c) is 2 (d) is 3 15. Let z be a complex number such that the
principal value of argument, arg z > 0.
9. The general solution of the differential
 x Then, arg z − arg(− z) is
 x π
equation 1 + e y  dx + 1 −  e x / y dy = 0 is (a) (b) ± π (c) π (d) − π
   y 2
 
(C is an arbitrary constant) 16. The general value of the real angle θ, which
x x satisfies the equation,
(a) x − ye = C y
(b) y − xe = C y (cos θ + i sin θ) (cos 2θ + i sin 2θ) ...
x x (cos nθ + i sin nθ) = 1 is given by, (assuming k
(c) x + ye y = C (d) y + xe y = C is an integer)
2 kπ 4kπ
dy (a) (b)
10. General solution of (x + y)2 = a 2 , a ≠ 0 is n+2 n(n + 1)
dx 4kπ 6kπ
(c) (d)
(C is an arbitrary constant) n+1 n(n + 1)
x y
(a) = tan + C (b) tan xy = C
a a 17. Let a , b, c be real numbers such that
y+C x+ y a + b + c < 0 and the quadratic equation
(c) tan( x + y) = C (d) tan =
a a ax 2 + bx + c = 0 has imaginary roots. Then,
11. Let P(4 , 3) be a point on the hyperbola (a) a > 0, c > 0 (b) a > 0, c < 0
x 2
y 2 (c) a < 0, c > 0 (d) a < 0, c < 0
2
−= 1. If the normal at P intersects the
a b2 18. A candidate is required to answer 6 out of
X -axis at (16, 0), then the eccentricity of the 12 questions which are divided into two parts
hyperbola is A and B, each containing 6 questions and
5 he/she is not permitted to attempt more than
(a) (b) 2 (c) 2 (d) 3 4 questions from any part. In how many
2
different ways can he/she make up his/her
12. If the radius of a spherical balloon increases choice of 6 questions?
by 0.1%, then its volume increases (a) 850 (b) 800 (c) 750 (d) 700
approximately by
(a) 0.2% (b) 0.3% (c) 0.4% (d) 0.05%
19. There are 7 greeting cards, each of a different
colour and 7 envelopes of same
13. The three sides of a right angled triangle are 7 colours as that of the cards. The number
in GP (geometric progression). If the two of ways in which the cards can be put in
acute angles be α and β, then tanα and tanβ envelopes, so that exactly 4 of the cards go
are into envelopes of respective colour is,
5+1 5 −1 5+1 5 −1 (a) 7C 3 (b) 2.7 C 3 (c) 3!4 C 4 (d) 3!7 C 3 4C 3
(a) and (b) and
2 2 2 2
1 5 2 20. 7 2n + 16 n − 1 (n ∈ N) is divisible by
(c) 5 and (d) and
5 2 5 (a) 65 (b) 63 (c) 61 (d) 64

Page 10 of 46
21. The number of irrational terms in the 28. Let the relation ρ be defined on R as aρb if
84
 1 1 1 + ab > 0 . Then,
expansion of 3 8 + 5 4  is (a) ρ is reflexive only.
 
  (b) ρ is equivalence relation.
(a) 73 (b) 74 (c) 75 (d) 76 (c) ρ is reflexive and transitive but not symmetric
(d) ρ is reflexive and symmetric but not transitive.
22. Let A be a square matrix of order 3 whose all
entries are 1 and let I 3 be the identity matrix 29. A problem in mathematics is given to
of order 3. Then, the matrix A − 3 I 3 is 4 students whose chances of solving
1 1 1 1
(a) invertible (b) orthogonal individually are , , and . The probability
(c) non-invertible 2 3 4 5
(d) real Skew Symmetric matrix that the problem will be solved at least by
one student is
23. If M is any square matrix of order 3 over ú 2 3
(a) (b)
and if M′ be the transpose of M, then 3 5
adj(M ′) − (adj M)′ is equal to (c)
4
(d)
3
(a) M (b) M′ 5 4
(c) null matrix (d) identity matrix
30. If X is a random variable such that σ(X ) = 2.6,
5 5 x x  then σ(1 − 4 X ) is equal to
 
24. If A = 0 x 5 x  and| A2 | = 25, then| x | is (a) 7.8 (b) − 10.4 (c) 13 (d) 10.4
0 0 5 
 − sin x
31. If e −e
sin x
− 4 = 0, then the number of
equal to real values of x is
1
(a) (b) 5 (c) 52 (d) 1 (a) 0 (b) 1
5
(c) 2 (d) 3
25. Let A and B be two square matrices of order 3
32. The angles of a triangle are in the ratio
and AB = O3 , where O3 denotes the null
2 : 3 : 7 and the radius of the circumscribed
matrix of order 3. Then,
circle is 10 cm. The length of the smallest
(a) must be A = O3 , B = O3 side is
(b) if A ≠ O3 , must be B ≠ O3
(a) 2 cm (b) 5 cm
(c) if A = O3 , must be B ≠ O3
(c) 7 cm (d) 10 cm
(d) may be A ≠ O3 , B ≠ O3

26. Let P and T be the subsets of k , y-plane 33. A variable line passes through a fixed point
(x1 , y1) and meets the axes at A and B. If the
defined by rectangle OAPB be completed, the locus of P
P = {(x , y) : x > 0, y > 0 and x 2 + y 2 = 1} is, (O being the origin of the system of axes).
x1 y
T = {(x , y) : x > 0, y > 0 and x 8 + y 8 < 1} (a) ( y − y1 )2 = 4( x − x1 ) (b) + 1 =1
x y
Then, P ∩ T is x2 y2
(a) the void set φ (b) P (c) x2 + y2 = x12 + y12 (d) 2 + 2 = 1
(c) T (d) P − T C 2 x1 y1

34. A straight line through the point (3, − 2) is


x2
27. Let f : R → R be defined by f (x) = x 2 − inclined at an angle 60º to the line 3 x + y = 1.
1 + x2 If it intersects the X -axis, then its equation
for all x ∈ R. Then, will be
(a) f is one-one but not onto mapping (a) y + x 3+2+ 3 3=0
(b) f is onto but not one-one mapping (b) y − x 3+2+ 3 3=0
(c) f is both one-one and onto (c) y − x 3 −2 −2 3=0
(d) f is neither one-one nor onto (d) x − x 3+2−3 3=0

Page 11 of 46
35. A variable line passes through the fixed point 42. P is the extremity of the latusrectum of
(α , β). The locus of the foot of the ellipse 3 x 2 + 4 y 2 = 48 in the first quadrant.
perpendicular from the origin on the line is The eccentric angle of P is
π 3π π 2π
(a) x2 + y2 − α x − β y = 0 (a) (b) (c) (d)
8 4 3 3
(b) x2 − y2 + 2α x + 2β y = 0
(c) αx + βy ± (α 2 + β 2 ) = 0 43. The direction ratios of the normal to the
x 2
y 2 plane passing through the points (1, 2 , − 3),
(d) + =1 x − 2 y +1 z
α 2
β 2 (− 1, − 2 , 1) and parallel to = = is
2 3 4
36. If the point of intersection of the lines (a) (2, 3, 4) (b) (14, − 8, − 1)
2 ax + 4 ay + c = 0 and 7 bx + 3 by − d = 0 lies in (c) (− 2, 0, − 3) (d) (1, − 2, − 3)
the 4 th quadrant and is equidistant from the 44. The equation of the plane, which bisects the
two axes, where a , b, c and d are non-zero line joining the points (1, 2, 3) and (3, 4, 5)
numbers, then ad : bc equals to at right angles is
(a) 2 : 3 (b) 2 : 1 (c) 1 : 1 (d) 3 : 2 (a) x + y + z = 0 (b) x + y − z = 9
(c) x + y + z = 9 (d) x + y − z + 9 = 0
37. A variable circle passes through the fixed
point A(p , q) and touches X -axis. The locus of 45. The limit of the interior angle of a regular
the other end of the diameter through A is polygon of n sides as n → ∞ is
(a) ( x − p)2 = 4qy (b) ( x − q )2 = 4 py π 3π 2π
(a) π (b) (c) (d)
(c) ( y − p) = 4qx2
(d) ( y − q ) = 4 px
2 3 2 3

46. Let f (x) > 0 for all x and f ′(x) exists for all x.
1 3
38. If P(0 , 0), Q(1, 0) and R  ,  are three given If f is the inverse function of h and
2 2  1
h′ (x) = . Then, f ′(x) will be
points, then the centre of the circle for which 1 + log x
the lines PQ, QR and RP are the tangents is
(a) 1 + log(f( x)) (b) 1 + f( x)
 1 3  1 1  (d)  1 , − 1
(a)  ,
1 1 (c) 1 − log(f( x)) (d) log f( x)
 (b)  ,  (c)  ,   
2 4  2 4  2 2 3 2 3
47. Consider the function f (x) = cos x 2. Then,
x 2
y 2 (a) f is of period 2 π (b) f is of period 2 π
39. For the hyperbola − = 1, which (c) f is not periodic (d) f is of period π
cos α sin α 2 2

of the following remains fixed when α varies? 48. lim (e x + x)1/ x


x → 0+
(a) directrix (b) vertices
(c) foci (d) eccentricity (a) Does not exist finitely (b) is 1
(c) is e 2 (d) is 2
40. S and T are the foci of an ellipse and B is the
end point of the minor axis. If STB is 49. Let f (x) be a derivable function, f ′ (x) > f (x)
equilateral triangle, the eccentricity of the and f (0) = 0 . Then,
ellipse is (a) f( x) > 0 for all x > 0
1 1 1 2 (b) f( x) < 0 for all x > 0
(a) (b) (c) (d) (c) no sign of f( x) can be ascertained
4 3 2 3
(d) f( x) is a constant function
41. The equation of the directrices of the
hyperbola 3 x 2 − 3 y 2 − 18 x + 12 y + 2 = 0 is
50. Let f :[1, 3] → R be a continuous function
13 6 that is differentiable in (1, 3) an
(a) x = 3 ± (b) x = 3 ± f ′ (x) = | f (x)|2 + 4 for all x ∈(1, 3). Then,
6 13
13 3 (a) f(3) − f(1) = 5 is true
(c) x = 6 ± (d) x = 6 ±
3 13 (b) f(3) − f(1) = 5 is false
(c) f(3) − f(1) = 7 is false
(d) f(3) − f(1) < 0 only at one point of (1, 3)

Page 12 of 46
Category II (Q.Nos. 51 to 65) 56. For any non-zero complex number z, the
Carry 2 marks each and only one option is minimum value of| z | + | z − 1| is
correct. In case of incorrect answer or any 1 3
(a) 1 (b) (c) 0 (d)
combination of more than one answer, 1/2 mark 2 2
will be deducted.
57. The system of equations
51. Let a = min{ x 2 + 2 x + 3 : x ∈ R} and λ x + y + 3z = 0
1 − cos θ n
2 x + µy − z = 0
b = lim
θ→ 0 θ2
. Then ∑ a r bn − r is 5x + 7 y + z = 0
r=0
has infinitely many solutions in R. Then,
2n + 1 − 1 2n + 1 + 1
(a) (b) (a) λ = 2, µ = 3 (b) λ = 1, µ = 2
3⋅2n 3⋅2n
n+1 (c) λ = 1, µ = 3 (d) λ = 3, µ = 1
4 −1 1
(c) (d) (2 n − 1)
3⋅2n 2 58. Let f : X → Y and A , B are non-void subsets
52. Let a > b > 0 and I (n) = a1/ n − b1/ n, of Y , then (where the symbols have their
usual interpretation)
J (n) = (a − b) 1/ n
for all n ≥ 2, Then
(a) f − 1( A) − f − 1(B) ⊃ f − 1( A − B) but the opposite
(a) I(n) < J(n) (b) I(n) > J(n) does not hold.
(c) I(n) = J(n) (d) I(n) + J(n) = 0
(b) f − 1( A) − f − 1(B) ⊂ f − 1( A − B) but the opposite
$ γ$ be three unit vectors such that
53. Let α$ , β, does not hold.
1 (c) f − 1( A − B) = f − 1( A) − f − 1(B)
α$ × (β$ × γ$) = (β$ + γ$) where
2 (d) f − 1( A − B) = f − 1( A) ∪ f − 1(B)
α$ × (β$ × γ$) = (α$ ⋅ γ$)β$ − (α$ ⋅ β$)γ$ . If β$ is not parallel
59. Let S, T , U be three non-void sets and
to γ$ , then the angle between α$ and β$ is f : S → T , g : T → U be so that gof : s → U is
5π π π 2π surjective. Then,
(a) (b) (c) (d)
6 6 3 3 (a) g and f are both surjective
(b) g is surjective, f may not be so
54. The position vectors of the points A , B, C and
(c) f is surjective, g may not be so
D are 3 $i − 2 $j − k$ , 2 $i − 3 $j + 2 k$ , 5 $i − $j + 2 k$ and (d) f and g both may not be surjective
4 $i − $j − λk$ , respectively. If the points A , B, C  π
60. The polar coordinate of a point P is 2, −  .
and D lie on a plane, the value of λ is  4
(a) 0 (b) 1 (c) 2 (d) − 4 The polar coordinate of the point Q which is
55. A particle starts at the origin and moves such that line joining PQ is bisected
1 unit horizontally to the right and reaches perpendicularly by the initial line, is
1 π π π π
P1 , then it moves unit vertically up and (a)  2,  (b)  2,  (c)  − 2,  (d)  − 2, 
2  4  6  4  6
1
reaches P2 , then it moves unit horizontally
4 61. The length of conjugate axis of a hyperbola is
1 greater than the length of transverse axis.
to right and reaches P3 , then it moves unit Then, the eccentricity e is
8
vertically down and reaches P4 , then it moves 1
(a) = 2 (b) > 2 (c) < 2 (d) <
1 2
unit horizontally to right and reaches P5
16 x  q
and so on. Let Pn = (x n , y n) and lim x n = α 62. The value of lim is
and lim y n = β. Then, (α , β) is n → ∞ x → 0+ p  x 
n→ ∞
[q ]
(b)  ,  (c)  , 1 (d)  , 3
4 2 2 4 (a) (b) 0 (c) 1 (d) ∞
(a) (2, 3)
 3 5 5  3  p

Page 13 of 46
63. Let f (x) = x 4 − 4 x 3 + 4 x 2 + c, c ∈ R. Then, 67. Two particles A and B move from rest along a
straight line with constant accelerations f
(a) f( x) has infinitely many zeros in (1, 2) for all c
and h, respectively. If A takes m seconds more
(b) f( x) has exactly one zero in (1, 2) if −1 < c < 0
than B and describes n units more than that
(c) f( x) has double zeros in (1, 2) if − 1 < c < 0
of B acquiring the same speed, then
(d) whatever be the value of c, f( x) has no zero in
(1, 2) (a) (f + h) m2 = fhn
(b) (f − fh)m2 = fhn
64. The graphs of the polynomial x 2 − 1 and cos x
1
intersect (c) (h − f )n = fhm2
2
(a) at exactly two points 1
(b) at exactly 3 points (d) (f + h) n = fhm2
2
(c) at least 4 but at finitely many points.
(d) at infinitely many points. 68. The area bounded by y = x + 1 and y = cos x
10 and the X -axis, is
65. A point is in motion along a hyperbola y = 3
x (a) 1 sq unit (b) sq unit
so that its abscissa x increases uniformly at a 2
rate of 1 unit per second. Then, the rate of 1 1
(c) sq unit (d) sq unit
change of its ordinate when the point passes 4 8
through (5, 2) 69. Let x1, x 2 be the roots of x 2 − 3 x + a = 0 and
1
(a) increases at the rate of unit per second x 3 , x 4 be the roots of x 2 − 12 x + b = 0 .
2
1 If x1 < x 2 < x 3 < x 4 and x1 , x 2 , x 3 , x 4 are in
(b) decreases at the rate of unit per second
2 GP, then ab equals
2
(c) decreases at the rate of unit per second 24
5 (a) (b) 64
5
2
(d) increases at the rate of unit per second (c) 16 (d) 8
5
1 − i cos θ
70. If θ ∈ ú and is real number, then
Category III (Q. Nos. 66 to 75) 1 + 2 i cos θ
Carry 2 marks each and one or more option(s) θ will be (when I : Set of integers)
π 3nπ
is/are correct. If all correct answers are not (a) (2 n + 1) , n ∈ I (b) , n∈I
marked and also no incorrect answer is marked 2 2
then score = 2 × number of correct answers (c) nπ, n ∈ I (d) 2nπ, n ∈ I
marked ÷ actual number of correct answers. If  3 0 3
any wrong option is marked or if any combination  
71. Let A = 0 3 0 . Then, the roots of the
including a wrong option is marked, the answer  3 0 3
will considered wrong, but there is no negative  
marking for the same and zero marks will be equation det(A − λ I 3) = 0 (where I 3 is the
awarded. identity matrix of order 3) are
(a) 3, 0, 3 (b) 0, 3, 6
1
(c) 1, 0, − 6 (d) 3, 3, 6
66. Let I n = ∫ x tann −1
x dx . If a nI n + 2 + bnI n = cn
0 72. Straight lines x − y = 7 and x + 4 y = 2
for all n ≥ 1, then intersect at B. Points A and C are so chosen
(a) a1, a2 , a3 are in GP on these two lines such that AB = AC . The
(b) b1, b2 , b3 are in AP equation of line AC passing through (2 ,− 7) is
(c) c1, c 2 , c 3 are in HP
(a) x − y − 9 = 0 (b) 23 x + 7 y + 3 = 0
(d) a1, a2 , a3 are in AP
(c) 2 x − y − 11 = 0 (d) 7 x − 6 y − 56 = 0

Page 14 of 46
73. Equation of a tangent to the hyperbola (c) If g (a) = 0 = g (b ), the equation g ′( x) + kg ( x) = 0
5 x − y = 5 and which passes through an
2 2 is solvable in (a, b ), k ∈ú
(d) If g (a) = 0 = g (b ), the equation g ′( x) + kg ( x) = 0
external point (2, 8) is
may not be solvable in (a, b ), k ∈ú.
(a) 3 x − y + 2 = 0 (b) 3 x + y − 14 = 0
(c) 23 x − 3 y − 22 = 0 (d) 3 x − 23 y + 178 = 0 x3
75. Consider the function f (x) = − sin π x + 3
74. Let f and g be differentiable on the interval I 4
and let a , b ∈ I , a < b. Then, (a) f( x) does not attain value within the interval [− 2, 2 ]
1
(a) If f(a) = 0 = f(b ), the equation f ′( x) + f( x)g ′( x) = 0 (b) f( x) takes on the value 2 in the interval [− 2, 2 ]
is solvable in (a, b ) 3
1
(b) If f(a) = 0 = f(b ), the equation f ′( x) + f( x)g ′( x) = 0 (c) f( x) takes on the value 3 in the interval [− 2, 2 ]
4
may not be solvable in (a, b ).
(d) f( x) takes no value p, 1 < p < 5 in the interval [− 2, 2 ].

Answers
Physics
1. (b) 2. (b) 3. (d) 4. (d) 5. (d) 6. (b) 7. (b) 8. (c) 9. (d) 10. (c)
11. (c) 12. (a) 13. (c) 14. (c) 15. (c) 16. (b) 17. (a) 18. (d) 19. (a) 20. (a)
21. (d) 22. (d) 23. (*) 24. (c) 25. (c) 26. (d) 27. (a) 28. (d) 29. (c) 30. (c)
31. (a) 32. (a) 33. (a) 34. (a) 35. (c) 36. (b, d) 37. (b) 38. (a, d) 39. (a, c) 40. (b, d)

Chemistry
41. (c) 42. (a) 43. (d) 44. (b) 45. (a) 46. (b) 47. (c) 48. (b) 49. (d) 50. (b)
51. (c) 52. (d) 53. (c) 54. (c) 55. (c) 56. (a) 57. (b) 58. (d) 59. (a) 60. (b)
61. (d) 62. (b) 63. (a) 64. (b) 65. (d) 66. (b) 67. (d) 68. (d) 69. (d) 70. (b)
71. (d) 72. (d) 73. (a) 74. (d) 75. (a) 76. (a, b) 77. (a, b) 78. (b, c, d) 79. (a, c) 80. (d)

Mathematics
1. (b) 2. (b) 3. (d) 4. (b) 5. (a) 6. (d) 7. (d) 8. (c) 9. (c) 10. (d)
11. (b) 12. (b) 13. (b) 14. (b) 15. (c) 16. (b) 17. (d) 18. (a) 19. (b) 20. (d)
21. (b) 22. (c) 23. (c) 24. (a) 25. (d) 26. (a) 27. (d) 28. (d) 29. (c) 30. (d)
31. (a) 32. (d) 33. (b) 34. (b) 35. (a) 36. (b) 37. (a) 38. (c) 39. (c) 40. (c)
41. (a) 42. (c) 43. (b) 44. (c) 45. (a) 46. (a) 47. (c) 48. (c) 49. (a) 50. (b, c)
51. (c) 52. (a) 53. (d) 54. (d) 55. (b) 56. (a) 57. (c) 58. (c) 59. (b) 60. (a)
61. (b) 62. (a) 63. (b) 64. (a) 65. (c) 66. (b, d) 67. (c) 68. (b) 69. (b) 70. (a)
71. (b) 72. (a, b) 73. (a, c) 74. (a, c) 75. (d)

Page 15 of 46
Answer with Explanations
Physics
1. (b) According to the question, 3. (d) As we know that de-Broglie wavelength is
h h
given as λ = = …(i)
p mv
n
e0 e where, h = Planck constant
θ θ p = momentum of particle
v = velocity of particle
and m = mass of the particle.
We know that incident ray, reflected ray and
normal lie in the same plane. Eq. (i) can be written as,
and angle of incidence = angle of reflection h h
λ= = [QKE = qv]
$ will be along the angle bisector of e$ and
Therefore n 2m(KE) 2mqv
− e$ 0 , where, KE = Kinetic energy of particle
$ = e + (− e 0)
$ $ 1
i.e. n …(i) Hence, λ ∝
|e$ − e$ 0 | m
[QBisector will along a vector dividing in same λ proton melectron
ratio as the ratio of sides forming that angle] Now, =
λ electron mproton
$ is a unit vector where|e$ − e$ 0 | = OC
But n
= 2OP = 2|e$ | Given, mass of proton, mproton = 2000 melectron
cosθ = 2cosθ λ proton 1
=
Substituting this value in Eq. (i), we get λ electron 2000
$ = e − e0
$ $
n λ proton 1 λe
2cosθ = ⇒ λp =
λ electron 20 5 20 5
e$ = e$ 0 + (2cosθ) n $
e$ = e$ 0 − 2(n$ ⋅ e$ 0)n $ ⋅ e$ 0 = − cosθ]
$ [Qn 4. (d) According to the Bohr’s atomic model,
nh
2. (b) According to the question, Angular momentum, L = mvr = …(i)
α-particle β-particle

X → Y → Z(say) Since, angular velocity ω =
v
According to Rutherford Soddy law of radioactive r
decay, the rate of decay of radioactive along at any ⇒ v = rω
instant is proportional to the number of atoms From Eq. (i), we get
present at that instant nh
dN m(rω) r =
= − λN 2π
dt nh
Since, λ = constant (decay constant) and mωr 2 =

N = number of atoms nh
Rate of decay of Y particle is given as, ω= …(ii)
2πmr 2
dN Y
λ X N X − λ Y NY = =0
dt Since, the radius of the electron in nth orbit of
[QDecay rate for β-particle become n2h2ε0
Bohr’s atomic model is given as, r = …(iii)
constant after some time] πmze 2
Given, rate of emission of β-particle = 107/h Squaring the Eq. (iii) and substituting its value in
Eq. (ii), we get
∴ λ X N X = λ Y N Y = 10 /h
7
nh(πmze)2 1
ω= ⇒ω ∝ 3
2πm(n4 h4 ε20) n

Page 16 of 46
5. (d) According to the question, 9. (d) According to the question, we can draw the
following diagram
1 kΩ
A

10 V 1 kΩ 6V
T T

B m1
a
m2 a
Input voltage VS = 10V m1g
Source resistance RS = 1kΩ
m 2g
Zener diode voltage VZ = 6V
Q Breakdown voltage of zener diode is = 6V, and the Here, T is the tension in the string.
potential difference across the zener diode 5V. In equilibrium condition,
∴Current flow in zener diode I Z = 0 For mass m1 , T − m1 g = m1 a …(i)
For mass m2, m2 g − T = m2a …(ii)
6. (b) By using the de-Morgan’s law
After adding Eqs. (i) and (ii), we get
A + B = A⋅ B
m2 g − m1 g = m1 a + m2a
Hence, option (b) is the correct. (m2 − m1) g = (m1 + m2)a
(m − m1)
7. (b) As we know that, a= 2 g
Impulse, I = force (F) × small time interval (m1 + m2)
I = ma × t [Q F = ma] 10. (c) Given, Power (P) = constant
I = [M] [LT− 2] × [T] 1
Kinetic Energy (KE) = mv2
I = [M L T− 1 ] 2
Hence, the correct dimensional formula for impulse KE mv2
We know that, P = ⇒P=
is given by [M L T− 1 ] . ∆t 2
Q P = constant,
8. (c) Given,
Hence, velocity of the body v ∝ t …(i)
Maximum error in the measurement of mass = 0.3%
ds
Maximum error in the measurement of length As, Velocity v = …(ii)
= 0.2% dt
We know that, From Eqs. (i) and (ii), we get
ds
Error in density is given as, So, ∝ t
mass (m) m dt
Density, ρ = = Integrating the above equation w.r.t. time ()
t,
volume (V) L3
ds
where, L = side of cube ∫ dt ∝ ∫ t
Error in density is given as,
we get, displacement of the body s ∝ t 3/ 2
 ∆ρ  ∆m 3∆L
  = + Q Displacement s = at 3/ 2, where a is constant.
 ρ  m L
 ∆ρ  ∆m 3∆L  11. (c) According to the question,
or   × 100 =  +  × 100
 ρ   m L  (r2) v v(r1)
P
Substituting the given values, we get
 ∆ρ 
  = (0.3% + 3(0.2)%) = 0.3% + 0.6%
 ρ  max
O
∴ Maximum percentage error measurement of Representation of position vectors
of two particles (drawn from the point P)
 ∆ρ 
density   = 0.9%.
 ρ  max

Page 17 of 46
In two dimension, the position vectors r1 and r2 13. (c) Given, Decrement in the length = 1%.
represented as
Poisson’s ratio for material of the rod, σ = 0.2
1
r1 = vt $i − gt 2$j …(i) As we know that,
2
1 Volume, v = πr 2l ,[where, l is the length of the rod]
r2 = vt(− $i) − gt 2($j) …(ii)
∆V 2∆r ∆l
2 = + …(i)
Q We know that, when the two vectors are V r l
mutually perpendicular, i.e. − ∆D / D
Since, Poisson’s ratio, σ =
θ = 90º ∆L/ L
So, r1 ⋅ r2 = r1 r2 cos 90º − ∆r / r
=
r1 ⋅ r2 = 0 ∆L/ L
Substituting the values r1 and r2 in the above So, Eq. (i) can be written as,
relation, we get ∆V ∆l
1 1 = (1 − 2σ)
((vt)$i − gt 2$j) ⋅ (vt(− $i) − gt 2($j)) = 0 V l
2 2 ∆V  ∆l
1 24 × 100 =  × 100 × (1 − 2σ)
− v t + 4 g t = 0 (where, i ⋅ $i = $j ⋅ $j = k$ ⋅ k$ = 1)
2 2 $ V  l 
4 = − 1 × [1 − 2 × (0.2)]
1
v2t 2 = g2t 4 = − 1 × [1 − 0.4]
4
1 22 = − 0.6%
⇒ v = gt
2
Here, negative sign shows the decrement in the
4
1 volume.
∴Magnitude of velocity of the particles, v = gt
2 14. (c) When the spherical body falls with constant
We know that, separation distance between velocity, i.e. terminal velocity then the net force
particles at a time t becomes zero, i.e the weight of body is equal to
∆x = 2vt the buoyancy force.
2v 4v2 Hence, Fnet = 0
∆x = 2 × v × ⇒ ∆x =
g g 15. (c) According to the question,
Area of both bodies A and B = A
12. (a) According to the Kepler’s third law
Temperature of body A = 27ºC = 27 + 273 K
T2 ∝ r3
Temperature of body B = 177ºC = 177 + 273 K
where, T = time period of revolution
Now, by Stefan-Boltzmann law, thermal energy
r = radius
2 3 2/ 3
radiated per second by a body
T  r  r T  Q = σAT 4 ,
Now,  E  =  E  = E =  E 
 TP   rP  rP  TP  where, A = Area
2/ 3
rE  2π /ωE  Q T = 2π  T = temperature
= 
rP  2π /ωP   ω  and σ = Stefan-Boltzmam’s constant
2/ 3 So, the ratio of thermal energy radiated per second
rE  ωP  by A to that by B is
= 
rP  ωE  4
Q1 σ A  T1 
According to the question, ωP = 2ωE and rE = R =  
Q2 σA  T2 
2/ 3
R  2ωE  4
⇒ =  Q1  T1 
rP  ωE  Now, = 
Q2  T2 
R
= (2)2/ 3 4
rP Q1  273 + 27 
= 
R Q2  273 + 177 
rP = = R(2)− 2/ 3
(2)2/ 3 Q1  300 
4
= 
Q2  450 

Page 18 of 46
4
Q1  2
=   =  
16 Similarly, in branch CD,
Q2  3  81  CCD = 10 µF
Ratio of thermal energy radiated per second Now, C AB and CCD are connected in parallel
combination. Hence, the equivalent capacitance of
Q1 : Q2 = 16 : 81 the circuit,
16. (b) For a gas at temperature T, the internal energy, Ceq = 10 + 10 = 20 µF
f We know that, charge Q = CeqV
U= µRT
2 Q = 20 × 10− 6 × 150 V
where, f = degree of freedom Amount of charge stored Q = 3 × 10− 3C
f
⇒ Change in energy, ∆U = µR ∆T …(i)
2 19. (a) The clock diagram is as shown below
Also, as we know for any gas heat supplied at +Q
+Q +Q
constant volume 12
11 1
(∆QV) = µCV ∆T = ∆U …(ii) +Q
10 2
where, CV = molar specific heat at constant volume
r
From Eqs. (i) and (ii), we get +Q 9 3 +Q
1 C
CV = fR
2 8 4 +Q
+Q
For diatomic gas, degree of freedom, f = 5 7 6 5
5 +Q +Q
CV = R +Q
2
In the above diagram, charge + Q is not placed at
17. (a) Since, the whole system is isolated, this 10 h position.
means, there is no transfer of heat between the
So, net electric field strength at centre C,
system and the surrounding. Also, the right side
container is initially vacuum, so the gas could E net = E1 + E 2 + E 3 + E 4 + E 5 + E 6
easily rush there without any resistive force. As + E 7 + E 8 + E 9 + E11 + E12
the right side container has no temperature. Thus, ⇒ E net = E1 + E 2 + E 3 + E 4
the temperature of the ideal gas would remain + E 5 + E 6 + (− E1 − E 2 − E 3 − E 5 − E 6)
same even if it enters right side chamber. Q
⇒ E net = E 4 = , from centre towards the
Therefore, final temperature attained at the 4 πε0 r 2
equilibrium will be T. mark 10.
18. (d) The given circuit shows a balanced 20. (a) According to the question,
Wheatstone bridge. Now, the circuit becomes Q
20µF 20µF
C D r
d

θ θ E
x –q
A B
20µF 20µF d

Q
150 V Force experienced by the charge − q due to charge Q
In the branch AB, both capacitor are arranged in 2kQq  1 
F = − 2 cosθ … (i) where, k = 
the series combination. Hence, its equivalent r  4 πε 0
capacitance is given by x
1 1 1 2 1 From diagram, cosθ =
= + = = r
C AB 20 20 20 10
By substituting the value of cosθ in Eq. (i)
C AB = 10 µF

Page 19 of 46
F=−
2kQq x
⋅ Q Net magnetic field at the centre = 0
r2 r So, magnetic force Fmagnetic = q(v × B) = 0
2kQx
or F=− So, the magnetic force experienced by the particle
r3 when it passes through the centre is F = 0
2kQx Q r 2 = x 2 + d 2 
or F=−   23. (c) According to the question,
(x + d 2)3/ 2
2
 r = x 2 + d 2  P
For, x < < d, so x can be neglected
2

2kQx
F=− 3 3m 4m
d
So, the force developed by negative charge (− q) due
to the system of the charges as shown in the figure A B
is, 5m
− 4kQqx
F= µ0 × I
d3 Magnetic field due to first wire (A) is B1 =
2π × 3
⇒ F∝x
µ ×I
So, the forced developed by negative charge is Magnetic field due to second wire (B) is B2 = 0
directly proportional to the distance x. 2π × 4

21. (d) If velocity of particle, v is perpendicular to Net magnitude of magnetic field B = B12 + B22
magnetic field, B i.e. θ = 90º, then particle will µ 0I 1 1 µ 0I × 5
experience maximum magnetic force, i.e. B= + =
2π 9 16 2π × 3 × 4
Fmax = qvB. This force acts in a direction
5 µ 0I
perpendicular to the motion of charged particle. Magnetic field B = ×
Therefore the trajectory of the particle is a circle. 24 π
In this case path of charged particle is circular and 24. (c) As we know that, magnetic field due to a long
magnetic force provides the necessary centripetal wire at a distance x from it is given by
force,
µ I
mv2 B = 0 , where, I is the current flowing through
i.e. qvB = 2πx
r the wire β ∝ I
mv ∴Magnetic flux associated with the square loop,
⇒ Radius of path, r =
qB φ ∝β ∝ I
r=
p
[Q p = mv] Now, if the current increase, then φ also increases.
qB Direction of long wire will be q.
where, p = momentum of the particle This means, magnetic field due to induced
∴ r ∝ momentum current will be opposite to the existing magnetic
field, i.e. according to Lenz’s law,
Hence, option (d) is correct.
The induced current in the loop will be in the
22. (d) Given, electric current in circular wire = I anti-clockwise direction. Now,
Radius of wire = r
Charge on particle = q FAD
Speed of the particle = v
The given loop can be as shown in the figure below. D A
I/2 FAB
FCD

r1
I I
C B

FBC
r2
I/2

Page 20 of 46
Since, wires attract each other, if current flowing 5 7
=
through them is in same direction and repel each S S+ 3
other, if currents are in opposite direction.
5 (S + 3) = 7S
∴ Part CD of the loop will experience a force of
5 S + 15 = 7S
repulsion, whereas part AB will experience
attraction. Parts BC and AD will not experience any 2S = 15
force. Thus, the overall force will be a force of S = 7.5 Ω.
repulsion because AB is closer to straight. The force
between two current carrying conductors is 27. (a) Given, inductance of inductor, L = 60 mH
inversely proportional to the distance between = 60 × 10− 3 H
them
Phase difference between voltage and current in
1
F∝ L-R circuit, θ1 = 60º
r
Capacitance of capacitor, C = 0.5 µF = 0.5 × 10− 6F
Q r1 < r2
Phase difference between voltage and current in
So, FCD > FAB
R-C circuit, θ2 = 30º
Fnet = FCD − FAB
For L-R circuit,
Hence, the loop will moves away from the wire. X
tanθ1 = L
25. (c) According to the question, R
ωL
I1 R R I2 R R tanθ1 = …(i) [Q X L = ωL]
I=I1/8 R
I0 I1/2 I 1 /4 I 1 /8 I Similarly, for R-C circuit,
2R V 2R 2R 2R 2R
X
tanθ2 = C
R
Total resistance of the given circuit 1
…(ii) Q X C =
1 
Req = 2R tanθ2 = ωC
R  ωC 
Now, circuit
From Eqs. (i) and (ii), we get
V
2R=I tanθ1 ωL
= = ω2LC
tanθ2 R × 1
I0
ωCR
2R V 2R tan 60º
= ω LC
2
tan 30º
B 3
= ω2LC
V / 2R 1/ 3
∴ I= (Q I1 = V / 2R)
8 ω2LC = 3
So, current I in the circuit I =
V Q I = I1  3
  ω2 =
16R  8 LC
3
26. (d) According to the balanced condition of ω2 =
Wheatstone bridge, 60 × 10− 3 × 0.5 × 10− 6
In the first case, 3
ω2 =
P
=
R 30 × 10− 9
Q S
ω = 108
P 5
= …(i)
Q S ω = 104
In the second case As we know that,
P
=
7
…(ii) ω = 2πf
Q S+ 3 ω 104 1
f = = Hz = × 104 Hz
From Eqs. (i) and (ii), we get 2π 2π 2π

Page 21 of 46
28. (d) According to the question, we can draw the 30. (c) Given, initial frequency of light,
following diagram f1 = 4 × 1014 s− 1
f=0.05 m Final frequency of light, f2 = 5 × 1014 s− 1
Change in wavelength , ∆λ = λ1 − λ 2
c c
axis or ∆λ = −
0.2 m
f1 f2
3 × 108 3 × 108
= −
4 × 1014
5 × 1014
Convex lens
3 × 108  1 1 
Given, u = − 0.2 m and f = 0.05 m =  − 
1014  4 5
As we know that,
3 × 108 1
1 1 1
= − …(i) = ×
f v u 1014 20
1 1 1 = 1.5 × 10− 7 m
= +
v f u Now, we know that,
1 1 1 Angular width of central maxima,
= −
v 0.05 0.2 2λ 2∆λ
θ= or ∆θ =
1 100 10 d d
= −
5 2 2∆λ
v d=
1 1 ∆θ
= 20 − 5 ⇒ v = m
v 15 Here, ∆θ = 0.6 radian
Now, differentiating eq. (i), we get 2 × 1.5 × 10− 7
d= = 5 × 10−7 m
dv du v2 0.6
− 2 = − 2 ∴ dv = du 2
v u u 31. (a) We know that, energy stored in the capacitor
2
2
 1 
A max = A ×   × 
1 1 2
 =CE
 15  (− 0.2)  2
1 and energy supplied by the source of emf
A max = A × × 25
225 = CE 2
A ∴ Energy dissipated in resistance R
A max =
9 = Energy supplied by the source of emf E
Here A is in cm. − Energy stored in the capacitor
A
Hence, A max = × 10− 2 m 1
= CE − CE 2
2
9 2
29. (c) As we know that,  1 1
= 1 −  CE 2 = CE 2
λD  2 2
fringe width, β =
d
32. (a) Given, cross sectional area of nozzle
where, λ = wavelength of light 5
D = Distance of the screen from the slits A= × 10− 3 m2
21
d = separation between the slits 1
∆β ∆D ∆d and rate of heat transfer Q = = 10− 1 m3 / s
Now, × 100 = × 100 + × 100 10
β D d
Q Heat transfer Q = AV
According to the question, Q 10− 1 m3 / s
∆β ∴ V= = = 2 21 × 10 m/s
× 100 = 0.5 + (− 0.3) = 0.5 − 0.3 = 0.2% A 5
× 10− 3 m2
β 21
Hence, the fringe width increases by 0.2%.

Page 22 of 46
When it hits a rigid wall then maximum possible E
So, v = is the condition for which the particle
increase in temperature of water can be expressed B
as moves undeflected in its original trajectory.
1 2
mv = ms∆T …(i) 35. (c) In given, Series R - L - C circuit
2
Resistance R = 100 Ω
where, m = mass, s = specific heat of water
Inductance of Inductor, L = 20 mH = 20 × 10− 3H
= 1 cal / g = 4.2 × 103 J / kg
1250
and ∆T = increase in temperature Resonance frequency f = Hz
π
1 v2
From Eq. (i), ∆T = Source voltage VDC = 25V
2s
According to resonant frequency,
(2 21 × 101)2
= ω0 = 2πf0 =
1
2 × 4.2 × 103 LC
84 × 102 1
= or (2πf0) =
2
8.4 × 103 LC
= 1° C 2 1250 × 1250 1
or 4π =
π× π LC
33. (a) Maximum loss in K . E = Total K . E energy
1000
before collision of two ice blocks or C=
1250 × 1250 × 4 × 20
1 1
K . Emax loss = mv2 + mv2 = mv2
2 2 (QBy substituting L = 20 × 10− 3)
This maximum loss in kinetic energy will be equal or C = 8 × 10− 6F
to loss in heat to melt two ice block. We know that, Charge QS = CV
⇒ mv2 = (ms ∆ θ + mL)2 …(i) QS = 8 × 10− 6 × 25 = 0.2 mC
According to question, So, the amount of charge stored in each plate of
. × 105 J kg − 1
L = 336 capacitor is 0.2 mC.
S = 2100 J kg − 1 K − 1 36. (b,d) In photoelectric effect, if the incident light
∆θ = 8º C had a frequency less than a minimum frequency
From Eq. (i) v = 2(s ∆ θ + L) ν0 , then no electrons are ejected regardless of the
light’s amplitude. This minimum frequency is also
= 2(2100 × 8 + 336
. × 105) called the threshold frequency, and the value of ν0
depends on the metal. For frequency greater than
v = 840 ms −1
ν0 , electrons would be ejected from the metal.
Furthermore, the kinetic energy of the
34. (a) According to the question,
photoelectrons is proportional to the light
Charge on particle is = q frequency (ν).
Velocity of particle = v The relationship between photoelectron kinetic
Due to uniform electric field, energy T and light frequency ν is shown in graph
electric force on particle below
Felectric = qE …(i)
Due to a uniform magnetic field,
magnetic force on particle is given by, Light
kinetic
Fmagnetic = q(v × B) … (ii) energy
When v is perpendicular to E and B, which are (T)
mutually perpendicular to each other,
Felectric = Fmagnetic
v0
From Eqs. (i) and (ii) Light frequency (n) Hz
qE = qvB
Graph shows that kinetic energy T is linearly
E
∴ v= increasing with light frequency (ν).
B

Page 23 of 46
However, for a given photosensitive material and 39. (a, c) The circuit diagram is as shown below,
frequency of incident light, number of
photoelectrons emitted per second is directly r r r r r r
proportional to the intensity of incident light
i.e., Number of electrons emitted ∝ J. A B
Also, photoelectric emission is an instantaneous r r r
process.
According to question, when given pair of point in
37. (b) The equation of state for an ideal gas options are connected through a resistance R, then
undergoing adiabatic process is given as, equivalent resistance between point A and B (R AB)
TV γ −1 = constant remains unchanged. It is only possible when current
does not flows through resistance R and circuit become
Let the temperature after adiabatic compression Wheatstone bridge as shown in the figure below,
as given in the question be T then,
γ −1 C
T0 V γ − 1 = T  0 
V
 2 P Q
γ −1
⇒ T = T0 2
A G B
V0
Now, heat released at volume to achieve
2 R S
temperature T0 .
D
The net heat released can be determined by the
equation. P R
At the balanced condition, =
∆Q = µCV ∆T Q S
R
=µ × (T0 2 γ − 1 − T0) The current flow in CD branch will be zero.
γ −1 Now by checking each option,
µRT0 γ − 1 from option (a), circuit is,
= (2 − 1) (QµT0 R = p0 V0)
γ −1 2
p V
∴ Heat released = 0 0 (2 γ − 1 − 1) 2r 4r
γ −1
A R B
38. (a, d) Given, initial velocity of particle,
−1
u = 10 ms
r 2r
Range, R = 5 m
6
Gravitational acceleration,
g = 10 ms− 2 P R 2r r 1 1
Now,Q = ⇒ = ⇒ =
Q S 4r 2 r 2 2
As we known that,
Hence, option (a) is correct.
u2 sin 2θ
Range of projectile, R = From option (b), circuit is
g
3
(10)2 sin 2α
5=
10 3r 2r
5
sin 2α =
10 A R B
1
sin 2α = r 2r
2
α = 15º 6
or (90° − 15° = 75º) P R 3r r
Now, = ⇒ ≠
Q S 3r 2r

Page 24 of 46
So, option (b) is also incorrect. P R 4r r
Now, = ⇒ ≠
From option (c) circuit is, Q S 2r 2r
4 So, option (d) is also incorrect.
Hence, option (a) and (c) are correct.
4r 2r
40. (b, d) According to the question,
A R B B

r 2r
7 Metallic loop
P R 4r 2r 2 2
Now, = ⇒ = ⇒ =
Q S 2r r 1 1
So, option (c) is also correct. Given, A metallic loop is placed in uniform
From option (d), circuit is magnetic field B. Magnetic field B and metallic loop
4 are perpendicular to each other. Then, if metallic
loop is moved along B or rotated about its own axis,
4r 2r the net flux associated with it remains constant.
Thus, no emf will be induced in these cases.
A R B However, when the loop is squeezed to a smaller
area or rotated about one of its diameters. Then its
r 2r flux changes. Thus, emf is induced. So, option (b)
7 and (d) both are correct.

Chemistry
41. (c) 2, 2, 2-trichloroacetophenone reacts with 42. (a) When salicylic acid reacts with acetic anhydride
aqueous KOH to form potassium Salt of benzoic in the presence of conc.H 2SO 4 and heat, the product
acid, which undergoes hydrolysis to form benzoic will be (Q) acetyl salicylic acid (C9 H 8O 4).
acid as the final product. CO2H CO2H
acetic anhydride
O O
Conc.H2SO4 (cat) Heat
C C HO H3COCO
(i) aq KOH 'Q'
CCl3 OH
(C9 H8 O4)
(ii) H3O+
Mechanism In presence of acid anhydride,
2, 2, 2-trichloro Benzoic acid nucleophilic acyl substitution reaction takes place.
acetophenone P O O O

Mechanism C C C
OH H 3C O CH3
+
O O
Acetic anhydride
HO (C4 H6 O3)
C C
CCl3 OK Salicylic acid
+ aq KOH + CHCl3 (C7 H6 O3) Conc. H2SO4 ∆
H3O+ O
2, 2, 2-trichloro
C
acetophenone O OH O
C O +
OH C H
H3C O
C
'P'
Benzoic acid O CH3
Acetyl salicylic acid Acetic acid
The product of the above reaction P is benzoic acid. (C9 H8 O4) (C2 H4 O2)
Thus, option (c) is correct answer. Aspirin
So, the option (a) is correct answer.

Page 25 of 46
0.5 / 15
43. (d) When cyclopentanol on reaction with NaH ∴ pH = pK a + log ⇒ pH = pK a
followed by CS2 and CH 3I produces a xanthate. 0.5 / 15
OH The value of pK a is the lowest for the mixture of
NaH CH 3COOH and CH 3COONa (acidic buffer).
Xanthate
CS2/CH3I Therefore, it has lowest pH.
Cyclopentanol
Hence, pH value for option (c) is the lowest.
Mechanism 48. (b) Given, for two first order reactions.
–+ A → B, k = 0.693 min− 1 …(1)
OH ONa
NaH S=C=S and A → C, t1 / 2 = 0.693 min …(2)
For first order reaction
Cyclopentanol S S 0.693
–+
t1 / 2 = ,
O—C—SNa O—C—S— CH3 k
CH3-I
+ NaI Thus, for the reaction A → B
0.693 0.693
Xanthate t1 / 2 = = = 1.0 min
k 0.693
So, the option (d) is correct answer.
Also, for first order reaction, lower the t1 / 2 value,
44. (b) CH 3  C Cl hydrolyses to form CH 3COOH faster is the reaction.
 1
Q t1 / 2 ∝
O k
even at 25ºC, which subsequently reacts with 0.693
NaHCO 3 (sodium bicarbonate) present in the and k for A → C is = 1 min− 1
0.693
same medium to form carbon dioxide (CO 2).
Thus, reaction (2) is faster than reaction (1) or
The reaction involved is given below. reaction (1) is slower than reaction (2).
CH 3COCl + NaHCO 3 → CH 3COONa + HCl + CO 2 Hence, option (b) is the correct answer.
Thus, the option (b) is correct answer.

45. (a) In BH −4 indicated atom is not a nucleophilic


49. (d) For the given relation H 2O()l - H 2O (v).
↑ (i) At equilibrium ∆G = 0
site because there is no lone-pair on ‘B’ atom. (ii) Entropy change, i.e. ∆S is the measurement of
Hence, the option (a) is correct answr. randomness. It increases in vapour-state,
i.e. ∆S = (+) ve or ∆S > 0
46. (b) One millimole = 0.001 mol (iii) When liquid H 2O changes into vapour H 2O, it
i.e.1 × 10− 3 mol. requires heat energy. Thus, ∆H > 0
Hence, correct set is
Q 96500 C is required for 1 mol of electrons.
∆G = 0, ∆H > 0 and ∆S > 0
193 × 1
∴ 193 C give = 0.002 mol, i.e. 2 × 10− 3 mol of So, option (d) is the correct answer.
96500
electrons. 50. (b) Dimensions of  ab2  are shown below.
0.002 V 
Thus, value of n in M n + is = =2
0.001 a
× b =pressure ×volume
Hence, option (b) is the correct answer. V2
force
47. (c) Key Point = × volume
area
For buffer solutions,
MLT−2
pH = pK a + log
[Salt] = × L3 = ML2T−2
[Acid] / [Base] L2
Moles ofCH 3COOH = 5mL × 0.01 m mol = 0.5 m mol The units of energy (joule) is,
Moles of CH 3COONa = 10 mL × 0.05 m mol 1 J = 1 kg m2s−2 = ML2T−2
= 0.5 m mol Therefore, the dimensions of  2  is same as that
ab
Total volume of solution = 5 + 10 = 15mL V 
of energy. So, the option (b) is correct.

Page 26 of 46
51. (c) In the equilibrium H 2 + I 2 - 2HI, if at a In the crystalline solid MSO 4 ⋅ nH 2O, the value of
n is 5.
given temperature, the concentrations of the
reactants are increased, the value of the Hence, option (c) is correct answer.
equilibrium constant K C will remain the same 56. (a) Given, mass of gas (W) = 7.5 g
because equilibrium constant does not depend on
the molar concentration of reactants. Volume of gas at STP (V) = 56. L
V(L) W
The option (c) is correct answer. Q Moles of gas at STP = =
22 . 4(L) M
52. (d) Electrolysis of copper sulphate solution using
Cu-electrodes, Where M = molar mass of gas.
Ions present : Cu 2 + , H+ , SO 24 − , OH − W × 22 . 4
∴ M=
V
At cathode Cu 2 + (aq) + 2e − → Cu (s)
7.5 × 22.4
At anode Cu(s) − 2e − → Cu 2 + (aq) = = 30.00 g mol −1
56
.
So, the option (d) is correct answer.
Among the given options
53. (c) The three quantum numbers n, l and m that Molar mass(M) of
describe an orbital has integer values of 0, 1, 2, 3 (a) NO = 14 + 16 = 30.00 g mol −1
and so on.
(b) N 2O = 28 + 16 = 44.00 g mol −1
Name Symbol Range of Values (c) CO = 12 + 16 = 28.00 g mol −1
Principal quantum n 1≤ n (d) CO 2 = 12 + 32 = 44.00 g mol −1
number
Q Molar mass of NO = 30 g mol −1 .
Azimuthal quantum l 0≤ l ≤ n − 1
number Thus, the given gas is NO.
Magnetic quantum m − l ≤ m≤ l Hence, (a) is the correct option.
number 57. (b) Let, initial concentration (a) = 100
The electronic arrangement of option (c) is absurd Given, half-life (t1 / 2) = 60 days
because To find, radioactivity,
n= 2 i.e. (a − x) after time T (180 days)
l=0 Q T = n × t1 / 2
m= 0 where, n = no. of half-lives
To sum up, where n = 2, l = 0, m = 0 180 = n × 60
Hence, for l = 0, value of m should be zero. 180
n= =3
60
54. (c) The quantity hv / K B corresponds to a 100 100
and (a − x) = n = 3 =
temperature. hv = 3 / 2 K B T 2 2 8
where, T = temperature (a − x) = 12 .5%
K B = Boltzmann constant Hence, (b) is the correct answer.
The percentage of H 2O in the solid is
58. (d) Given, 82 A
210
→ B → C → 82D
206
= (100 − 64) = 36%
hv 3 i.e. difference in mass no. between A and D is of 4
= T
KB 2 units i.e., mass no. is decreased by 4 units while
atomic no. remains the same, i.e. 82 for A and D.
3 hv
Since, is a constant, the value of corresponds Also,
2 KB
(i) On emission of each β-particle, mass no.
to temperature. Therefore option (c) is correct.
remains the same but atomic no. is increased
55. (c) Mass of H 2O = 36 × 250 = 90 g by one unit.
100 (ii) On emission of each α-particle, mass no. is
90
Moles of H 2O = = 5mol decreased by 4 units, while atomic no. is
18 decreased by 2 units.

Page 27 of 46
Thus, on emission of 2-β and 1-α particle, (ii) Smaller the size, more is value of electron affinity, i.e.
we get 82 D 206, more (−) ve will be the electron gain enthalpy.
−β −β −α On moving across a period, the size of atoms
i.e. 82 A
210
→ 83 A210 → 84 A210 → 82 A206 decreases due to increase in nuclear charge.
Thus, option (d) is the correct answer. (i) Electron configuration of the elements are
shown below.
59. (a) Key Point For similar electronic configuration of
C(Z = 6) = 1s 2 2s 2 2p 2
outer most shell, size will decide the value of ionisation
energy. More the size, lesser is the value of ionisation N(Z = 7) = 1s 2 2s 2 2p 3
energy. O(Z = 8) = 1s 2 2s 2 2p 4
Electronic configuration IInd I.E (ii) Due to half-filled electronic configuration of
Zn+ ion (Z = 30) = [Ar] 3d 10 4s1, (1734 kJ/mol). nitrogen (i.e. 3 electrons in 3p orbitals), it is
highly stable and has (+) ve value of electron
Cd + ion (Z = 48) = [Kr] 4d 10 5 s1, (1631 kJ/mol) gain enthalpy (i.e. about 30.9 kJ/mol).
+
Hg ion(Z = 80) = [Xe] 4f ⋅ 5d 14 10 1
6s (1809 kJ/mol) (iii) As size of ‘O’, is smaller than that of ‘C’, ‘O’
atom has higher (−) ve electron gain enthalpy
Q Size of Cd+ > Zn+ , thus it has lower IInd (about − 141.1 kJ / mol) than that of carbon
ionisation energy while due to lanthanoid effect (C-atom) [about − 122.3 kJ / mol].
(i.e., poor screening by 4 f and 5d electrons, Hg + has Hence, correct order is O > C > N and (b) is
higher IInd ionisation-energy. the correct option.
Hence, correct order is,
Zn > Cd < Hg and option (a) is the correct answer.
63. (a) Key point As the percentage of s character in a
bond increases, the bond angle also increases. Therefore,
60. (b) Key Point A compound having more ionic as the bond angle increases, the percentage of
character, has more melting point. p-character decreases.
Be and Ca belong to the same group and ionic In the given options,
character increases down the group. Ammonia (NH 3) has bond angle = 107.6º
Thus, BeCl 2 is less ionic than CaCl 2. This means and Phosphine (PH 3) has bond angle = 93. 5º
that melting point of CaCl 2 is higher than that of 1
BeCl 2. Q Bond angle ∝ and ∝ s-character
p - character
Also, Hg is a transition metal, its compound is less
ionic than BeCl 2. Therefore, order of melting point Bond angle of NH 3 > PH 3 due to lone-pair lone-pair
will be repulsion.
CaCl 2 > BeCl 2 > HgCl 2 Thus, the lone pair on NH 3 has less p-character.
or (ii) > (i) > (iii) Hence, option (a) is the correct answer.
Hence, option (b) is the correct answer. 64. (b) Chlorine bleach is NaOCl. The hypochlorite
ion in chlorine bleach dissociates to give nascent
61. (d) Key point The species which has one or more oxygen as shown below.
unpaired electrons have non-zero magnetic moment.
OCl − → [O] + Cl −
Electronic configuration of
Therefore, the reactive species in chlorine bleach is
Na+(Z = 11) = 1s 2 2s 2 2p 6
OCl − .
Mg (Z = 12) = 1s 2 2s 2 2p 6 3s 2 So, the option (b) is correct.
F − (Z = 9) = 1s 2 2s 2 2p 6 65. (d) Co-ordinate compound of Co (III) dissociates
Ar + (Z = 18) = 1s 2 2s 2 2p 6 3s 2 3p 5 into 3 ions in the solution means it has two
ionisable-ions out side the coordination sphere
As, Ar + has an unpaired electron, it has non-zero with oxidation state of cobalt (Co) = + 3
magnetic moment. Hence, (d) is the correct option.
In option (a)
62. (b) Key Point Name : Hexaammine cobalt (III) chloride
(i) Half-filled/fully-filled configuration have (−) ve or Formula : [Co(NH 3)6]Cl 3
low electron affinity i.e., (+) ve or high electron gain Q Oxidation state of Co = (+) 3
enthalpy.

Page 28 of 46
In option (b) anti-conformations can be interconverted by
Name : Pentaammine sulphatocobalt (III) chloride rotation around C2-C3 linkage.
Formula : [Co(NH 3)5(SO 4)]Cl CH3 CH3 CH3
CH3
Q Oxidation state of Co = + 3 H H H CH3
In option (c)
Name : Pentaamminechloridocobalt (III) sulphate H H H H H H
Formula : [Co(NH 3)5Cl]SO 4 H H
CH3 H
Q Oxidation state of Co = + 3
Anti- Gauche- Eclipsed-
In option (d) conformer conformer conformer
Name : Pentaamminechloridocobalt (III) chloride
Hence, the option (d) is correct answer.
Formula : [Co(NH 3)5Cl]Cl 2
QOxidation state of Co = + 3 70. (b) In halogen acids, as the size of halogen atom
increases, the bond between halogen and
Dissociation of this compound is shown below:
2+
hydrogen atom weakens. Therefore, the case with
[CO(NH 3)5Cl]Cl 2 → CO(NH 3)5Cl] + 2Cl − which the bond can be broken increases.
144424443
3 ions The correct order of the addition reaction rates of
This compound can give total 3-ions, as follows. halogen acids with ethylene is hydrogen iodide
(HI) > Hydrogen bromide (HBr)
one [Co(NH 3)5Cl]2 + and two Cl − ions.
> Hydrogen chloride (HCl)
Hence, option (d) is the correct answer. So, the option (b) is correct answer.
66. (b) In the Bayer’s process, the leaching of
alumina is done by using NaOH. 71. (d) Total number of isomeric linear dipeptides
which can be synthesised from racemic mixture of
Al 2O 3(s) + 2NaOH(aq)  
→
150º C, 35 atm
alanine are four (4), by (R) type and (S) type.
Bauxite ore
2Na[Al(OH)4 ] +3H 2O (Q Has two chiral centres)
Sodium i.e.
aluminate
(soluble in water) O
CH3 H
2Na[Al(OH)4 ] + CO 2 → Al 2O 3 ⋅ xH 2O + 2NaHCO 3 H 2N COOH C
Al 2O 3 ⋅ xH 2O(s) 1470
 
K
→ Al 2O 3(s) + xH 2O(g) H 2N COOH
Pure alumina H 3C H NH
So, the option (b) is correct answer. CH3 H

The possible isomers are


67. (d) Among the given options,
238 (RR), (SR), (RS) and (SS).
92 U is an isotope of uranium but cannot be used
Hence, total (4) isomeric linear dipeptides for
as a nuclear-fuel. U-238 is non-fissile, i.e., it does
alanine can be synthesised and option (d) is the
not undergo fission by thermal neutrons. The
correct answer.
energy released when U-238 absorbs a neutron is
insufficient to carry out nuclear fission. 72. (d) From the given graph
Hence, (d) is the correct option. (rate)1 / 2
Slope = =4
68. (d) The molecule in which a lone pair of electrons [A]
undergoes conjugation with an alternate double Q rate (r) = k[A]n
bond is said to be delocalised. The molecule that Where, k = rate constant
has delocalised lone-pair of electrons is,
[A] = concentration at time t (in min).
O—CH3 O—CH3 n = order.
According to the graph (n = 2)
rate
∴ k= = [4]2 = 16
So, option (d) is correct. [A]n
Hence, n = 2and
69. (d) The conformation of n-butane, commonly k = 16.0 dm3 mol − 1 min− 1
known as eclipsed, gauche and

Page 29 of 46
and option (d) is the correct answer. (b)
O O α
73. (a) Key Point Me
I2/KOH
CH3
According to Kirchhoff’s equation Ph Ph
(O) O
∆H( f ) = ∆H( i) + CP (∆T) OH
At constant pressure (2º alcohol)
∆Cp = O
Therefore, heat of formation (∆H f ) does not O O
ONa CI3
depend on temperature and value of ∆Cp ,
Ph + CHI3 Ph
i.e. when ∆Cp = 0 O
O
Hence, option (a) is the correct answer.

74. (d) A copper coin was electroplated with zinc Thus, options (a, b) are correct.
(Zn) and then heated at high temperature until
there is a change in colour. The resulting colour 77. (a, b)
will be golden. (a) In CrO 5 oxidation no. of Cr is + 6.
The golden colour is due to the zinc migrating
–2
through the copper to form the alpha-form of brass –1O O O –1
alloy (percentage of Cu > 65% and that of Zn <
35%). Cr

Zn + Cu → Brass –1O O –1
Thus, the option (d) is correct answer. (b) For the reaction (in ideal case)
75. (a) N 2 O4 (g) → 2NO2 (g)
OH Peracid
O
OH ∆H = ∆U + p. ∆V
Q ∆H = ∆U + ∆n g RT
RCO3H
Allyl
[O]
(C3H6O2) (Q p∆V = ∆n g RT)
alcohol
Where, ∆n g = (gaseous moles of product)
Oxidation of allyl alcohol with a peracid gives a
compound of molecular formula C3H 6O 2, which − (gaseous moles of reactant)
contains an asymmetric carbon atom. ∆n g = 2 − 1 = 1
The structure of the compound is Thus, ∆H > ∆U
O (c) pH of 0.1 N H 2SO 4 is less than of 0.1 HCl at
OH
25º is a wrong statement.
pH of H 2SO 4
So, the option (a) is correct answer.
Q pH = − log[H + ] = − log[10− 2]
76. (a, b) Haloform reaction with I 2 and KOH
Q (N = C × Z) and (Z = 2for H 2SO 4 )
(a)
O O pH = 2and pH of HCl
I Ph I /KOH
2 I
Ph pH = − log[10− 1 ] = 1
α CHI3
I Thus, pH of H 2SO 4 > pH of HCl.
I
O RT
(d) = 0.0591 at 25ºC is a wrong statement.
+ Ph—CH2—C—ONa F
Where, R = Gas constant = 8.314
T = Temperature = 273 + 25 = 298 K
F = Charge over one mole of electrons
= 96500C

Page 30 of 46
RT 8.314 × 298 • No reaction with water
Thus, = = 0.02567
F 96500 Thus, NH 4NO 3 can be used to label all three
2.303 RT beakers.
The correct value is = 0.0591
F (c) (NH 4)2CO 3 will evolve pungent smelling gas
Hence, only option (a) and (b) are correct. NH 3 with NaOH.
• Effervescence of CO 2 gas with con.H 2SO 4
78. (b,c,d) Among the given species
• No reaction with water.
(a) H 2O and F are weak field ligands and do not
cause pairing of electrons in the central metal Thus, (NH 4)2CO 3 can be used to label all three
atom. As CN is a strong field ligand, it causes beakers.
pairing of electrons in the central metal atom. Thus, options (a, c) are correct.
Now,
80. (d) O
Oxidation state of :
Me Me
(b) Fe in [Fe(H 2O)6] Cl 3 = (+) 3and electronic Et
CO2COOH Et CH2—C— OH
∆ (100ºC) ∗
configuration of Fe3 + ions is 3d 5 4s 0 O
O
Fe3 + =
–CO2 ∗
H—O—C C—OH CH (Me) COOH
3d5 4s0 Me Chiral

(b)
i.e. have 5 unpaired electrons
Me Me
(c) Fe in K 3 [FeF6] Et CH2COOH Et CH2COOH
O ∆ (100ºC) ∗
Oxidation no. of Fe = (+) 3
–CO2
electronic configuration of Fe3 + is 3d 5 4s 0 HO—C CO2H CH2(COOH)
3+ 3d5 4s0 Chiral
Fe =
(c)
Me Me
i.e. have 5 unpaired electrons. Et CO2H Et H
∆ (100ºC) ∗
(d) Oxidation state of Mn in K 4 [MnF6] = (+) 2and
electronic configuration of Mn2 + is 3d 5 4s 0 Me
–CO2
Me
2+ O O
Mn =
Chiral
3d5 4s0
(d)
H H
Et CO2H Et H
i.e., have 5 unpaired electrons. ∆ (100ºC) ∗
Hence, options (b) (c) and (d) are the correct –CO2
options. O Me O Me

79. (a, c) Achiral


(a) NH 4NO 3 will evolve NH 3 pungent smelling gas Thus, compound (d), capable of producing achiral
with NaOH. compound on heating at 100ºC.
• NO 2 brown gas with con.H 2SO 4

Page 31 of 46
Mathematics
π /4
1. (b) We have,  µ sin x 
lim x ln x n
4. (b) Let I = ∫  λ |sin x | +
 1 + cos x
+ γ  dx

π
x → 0+ −
4
ln x π /4 π /4
= lim − n sin x
x → 0+ x = ∫ (λ|sin x | + γ) dx + µ ∫ + cos x
dx
− π /4 π1
1 −
4
= lim x [using L’Hospital’s rule] sin x
x → 0 + − nx
− n −1 Let f (x) =
1 + cos x
−1  1  sin(− x) − sin x
= lim =0 Q x − n = 0, when x = 0 ⇒ f (− x) = = = − f (x)
x→ 0 +
nx − n   1 + cos(− x) 1 + cos x

2. (b) Let I = ∫ cos x log  tan x  dx ∴ f (x) is an odd function.


 2 π /4
 sin x 
= log  tan  ⋅ sin x − ∫ sin x ⋅
1 x 1
So, ∫   dx = 0
π  1 + cos x 
x
⋅ sec2 ⋅ dx
 2 tan
x 2 2 −
4
2 π /4

= sin x log  tan  − ∫ sin x


x 1
dx
∴ I= ∫ λ |sin x | + γ) dx
 2 x
2sin cos
x −
π
4
2 2
∴ I is independent of µ.
 x
= sin x ⋅ log  tan  − ∫
sin x
dx
 2 sin x 5. (a) We have,
= sin x ⋅ log  tan  − ∫ 1 dx 1  sin2 t 
x a a
2
 2 ∫ e
lim dt − ∫ esin t dt 
x→ 0 x  
y x + y 
= sin x log  tan  − x + c
x
 2 1  
a x + y
2 2
= lim  ∫ esin t dt + ∫ esin t dt 
∴ f (x) = c − x x→ 0 x  
y a 
 
3. (d) Let I = ∫ cos2tan− 1 1 − x  dx 1
x + y
2
= lim ∫ esin t
 1+ x dt
x→ 0 x
y
Put x = cos 2θ ⇒ dx = − 2sin 2θ dθ
x + y
 1 − cos 2θ 
2
∴ I = ∫ cos2 tan− 1  (− 2sin 2θ) dθ ∫ esin t dt
 1 + cos 2θ 
= lim y
x→ 0 x
 2sin θ 
2
= − 2∫ cos2 tan− 1  sin 2θ dθ esin
2
( x + y)
(1 + 0) − 0 2
 2cos2 θ  = lim = e sin y
x→ 0 1
= − 2 ∫ cos{2tan− 1 (tanθ)} sin 2θ dθ x
6. (d) Let I = ∫ 22 ⋅ 2x dx
= − 2∫ cos 2θ sin 2θ dθ
Put t = 2x
= ∫ cos 2θ d(cos 2θ)
⇒ dt = 2x log 2 dx
cos 2θ2
2t
= + C
2
∴ I= ∫ log 2 dt
x2 x
= + C 2t 22
2 = 2
+ C= + C
(log 2) (log 2)2
Which is an equation of parabola.
1
∴ A=
(log 2)2

Page 32 of 46
1
 x 2015 1  1 1
= 3∫
7. (d) Let I = ∫  e| x |(x 2 + cos x) + | x|
e 
dx 0 1 + 3x
dx
−1
1
1
x 2015 1
1 = 3∫ (1 + 3x)− 1 / 2 dx
= ∫ e| x |(x 2 + cos x)
dx + ∫ e| x |
dx 0
1
−1 −1  2 1 + 3x 
= 3 
x 2015 1  3 0
Let f (x) = and g(x) =
| x|
e (x 2 + cos x) e| x | = [4 − 2]
(− x)2015 =2
Again, f (− x) =
e| − x |((− x)2 + cos(− x))  x 
− x 2015 9. (c) 1 + e y  dx + 1 − x  e x / y dy = 0
= = − f (x)  y
e| x |(x 2 + cos x)  
1 1  x
and g(− x) = | − x | = | x | = g(x) ⇒ (1 + e x / y)dx = − e x / y 1 −  dy
e e  y
Q f (x) is odd function and g(x) is even function. dx − e x / y (1 − x / y)
⇒ = …(i)
1 1 1
dy (1 + e x / y)
∴ ∫ f (x) dx = 0 and ∫ g(x) dx = 2 ∫ g(x) dx This is homogeneous differential equation.
−1 −1 0
1 So, put x = vy
1
∴ I = 2∫ dx dx dv
e| x | =v+ y …(ii)
0 dy dy
1 1
= 2∫
1
dx = 2∫ e − x dx dv − e v (1 − v)
Therefore, v + y =
0
ex 0 dy 1 + ev
= − 2 [e − x ]10 [from Eqs. (i) and (ii)]
= − 2 [e − 1 − e 0 ] ⇒
dv − e v + ve v
y = −v
= − 2 (e − 1 − 1) dy 1 + ev
( − e − 1)
= 21 − e v + ve v − v(1 + e v)
=
1 + ev

8. (c) lim 3 1 + n
+
n
+
n
+ ... = − e v + ve v − v − ve v
n→ ∞ n
 n+ 3 n+ 6 n+ 9
dv − (e v + v)
n  ⇒ y =
+  dy 1 + ev
n + 3(n − 1) 
1 + ev dy
⇒ dv = − [variable separation]
3 n n n v + ev
= lim  + + y
n→ ∞ n
 n + 3× 0 n + 3×1 n + 3× 2
1 + ev dy
n n 
⇒ ∫ v + ev dv = − ∫ y
[by integration] …(iii)
+ + K+ 
n + 3× 3 n + 3(n − 1)  Let v + e v = t ⇒(1 + e v) dv = dt
3 n − 1 n  1 + ev dt
= lim ∑  ∴ ∫ v + ev dv = ∫ = log t = log(v + e v) …(iv)
n→ ∞ n
 r = 0 n + 3r  t

  ⇒ log(v + e v) = − log y + log C


3 
n −1
1 [from Eqs. (iii) and (iv)]
= lim  ∑ 
n→ ∞ n  r
 r = 0 1 + 3    ⇒ log(v + e v) + log y = log C
  n  
⇒ y(v + e v) = C
n −1
1 1 x 
= 3 lim ∑ ⇒ y + e x / y  = C
1 + 3 
n→ ∞ n r
r=0 y 
 n

Page 33 of 46
 x + ye x / y  11. (b) Given hyperbola equation
⇒ y  =C
 y  x 2 y2
H: − =1
⇒ x + ye x/ y
=C a2 b2
P(4, 3)lie on H
10. (d) (x + y)2 dy = a 2, a ≠ 0 16 9
dx ⇒ − =1
a2 b2
Let x + y = t
dy dt Normal equation at P(4, 3) for H
1+ = a 2 y1 (x − x1) + b 2 x1 (y − y1) = 0
dx dx

dy dt
= −1 ⇒ 3a 2(x − 4) + 4b 2(y − 3) = 0
dx dx Normal cuts the X-axis at (16, 0)
Therefore, t 2  − 1 = a 2
dt
⇒ 3a 2(16 − 4) + 4b 2(0 − 3) = 0
 dx 
⇒ 3a 2(12) + 4b 2(− 3) = 0
2 dt
⇒ t − t = a2
2
dx ⇒ 36a 2 − 12b 2 = 0

⇒ t2
dt
= a2 + t2 ⇒ 3a 2 − b 2 = 0
dx
⇒ 3a 2 = b 2
t2
⇒ dt = dx [variable separation] ∴ Eccentricity of the hyperbola is
a + t2
2

a2 + b2
t2 e=
⇒ ∫ (t 2 + a 2) dt = ∫ dx [integration] a
a + 3a 2
2
4a 2 2a
t2 + a2 − a2 = = = =2
⇒ ∫ (t 2 + a 2) dt = x + C′ a a a

[doing a 2 adding and subtracting] 12. (b) Let V be the volume of spherical ballon of
radius r.
 a2 
⇒ ∫ 1 − t 2 + a 2  dt = x + C′ Then, V =
4 3
πr
3
4π 
a2 log V = log 
1 dV 3
⇒  + 3log r ⇒ =0+
⇒ ∫ dt − ∫ t 2 + a 2 dt = x + C′  3 V dr r
dt 1 dV 3 1 ∆V 3
⇒ t − a2∫ = x + C′ ⇒ = ⇒ =
t2 + a2 V dr r V ∆r r
∆V ∆r
1 t  ⇒ =3
⇒ t − a2 tan− 1   = x + C ′ V r
 a  a  
∆V ∆r
x + y ⇒ × 100 = 3 × 100
⇒ (x + y) − a tan− 1   V r
 a  ∆V
⇒ × 100 = 3 × 01 . = 0.3
= x + C ′[on putting the value of t] V
y − C′ x + y
⇒ = tan− 1  
∴ Percentage increase in volume is 0.3%.
a  a 
13. (b) Let ∆ABC be a right angled triangle at B. Let
x+ y y − C′ 
⇒ = tan  ∠A and ∠C be α and β
a  a 
A
x+ y y + C
⇒ = tan 
a  a  α
[where, C = − C ′ is an arbitrary constant] a
ar2
y + C x + y
⇒ tan  =
 a  a
β
B ar C

Page 34 of 46
Since, sides are in GP so sides are a, ar, ar 2 15. (c) (Im)
Now, AC 2 = AB2 + BC 2
(z)
⇒ (ar 2)2 = a 2 + (a ⋅ r)2
⇒ a 2r 4 = a 2 + a 2r 2 θ
(Re)
⇒ r − r −1 = 0
4 2
–π+θ
1+ 5
⇒ r2 = [r 2 > 0] (–z)
2
5+1
⇒ r= ∴ arg(z) − arg(− z) =θ − (− π + θ )
2
BC ar = π
∴ tanα = =
AB a 16. (b) We have,
5+1 (cosθ + i sinθ) (cos 2θ + i sin 2θ) ...
⇒ r=
2 (cos nθ + i sin nθ) = 1
and tanβ =
AB
=
a 1
= ⇒ e iθ ⋅ e i( 2θ) ⋅ e i( 3θ) ⋅ ... e i( nθ) = 1
BC ar r
⇒ e iθ(1 + 2 + 3 + K + n)
=1
1 5 −1 i n( n + 1) θ
= =
5+1 2 ⇒ e 2 =1
n(n + 1)  n(n + 1) 
⇒ cos θ + i sin
2
 θ = 1 + 0i
 2   2 
14. (b) We have,
n(n + 1) 
1  1  ⇒ cos θ = 1
log 62 + = log 2  2x + 8  2 
2x  
  n(n + 1)
⇒ θ = 2kπ
1  1  2
⇒ 1 + log 32 + = log 2  2x + 8 4k
2x   ⇒ θ= π
  n(n + 1)
1
1
2x + 8 1+ 17. (d) Let f (x) = ax 2 + bx + c
⇒ =2 2x
3 ⇒ 1 =a+ b+ c<0
f ()
1 1 Again, f (x) has imaginary zeros. So, a < 0.
⇒ 2x + 8= 3⋅ 2⋅ 22x Also, f (0) = c. Since f (x) is downward parabola. So,
1 1 c < 0.
⇒ 2x + 8 = 6 ⋅ 22x Y
1
Let y= 22x
⇒ y2 + 8 = 6 y
X′ X
⇒ y2 − 6 y + 8 = 0
⇒ (y − 4) (y − 2) = 0 f (x)
⇒ y = 4, 2
1 1 Y′
⇒ 22x = 4 and 22x = 2
1 1 18. (a)
⇒ = 2 and =1 A B
2x 2x
1 1 4 2
⇒ x= ,
4 2 3 3
2 4

Page 35 of 46
∴ Total number of ways 1 1 1  3 0 0
= 6C4 × 6C2 + 6C3 × 6C3 + 6C2 × 6C4 = 1 1 1 −  0 3 0
   
6 × 5 6 × 5 6 × 5× 4 6 × 5× 4 1 1 1  0 0 3
= × + ×
2×1 2×1 3× 2×1 3× 2×1 − 2 1 
1
+
6×5 6×5
× =1 −2 1 
 
2×1 2×1  1 1 − 2
= 15 × 15 + 20 × 20 + 15 × 15 = [− 2(4 − 1) − 1(− 2 − 1) + 11
( + 2)]
= 225 + 400 + 225 = [− 2(3) − 1(− 3) + 1(3)]
= 850 = [− 6 + 3 + 3] = 0
19. (b) Required number of ways Now, det = 0
= 7C4 × D(3) So, matrix A − 3I 3 is non-invertible matrix.
 1 1 
= 7C3 × 3!1 − + 23. (c) Since, adj (M ′) = (adj M)′
1
− 
  1! 2! 3!   = adj (M ′) − (adj M)′
Q n Cr = n Cn − r , and  [Q adj (A′) = (adj A)′]
 
 D(n) = n!1 − 1 + 1 − 1 + K   = 0, a null matrix.
  1! 2! 3!  
24. (a) Given,
= 7C3 × 2 = 2(7 C3)  5 5x x
A = 0 x 5x 
20. (d) We have,  
 0 0 5 
72n + 16n − 1
∴ A2 = A ⋅ A
Put n = 1, we get
72 + 16 − 1  5 5x x  5 5x x
= 0 x 5x  0 x 5x 
= 49 + 15    
= 64  0 0 5   0 0 5 
which is divisible by 64.  25 25x + 5x 2 5x + 25x 2 + 5x 
 
 1 1 
84 =0 x2 5x 2 + 25 
21. (b) Given,  38 + 54  0 0 25 
   
 
84 − r r = 25(25x 2 − 0)
 1  1
here, Tr + 1 = 84 Cr  38   54  = 25(25x 2)
   
    Since, given that| A |2 = 25
 84 − r  r
Now, 25(25x 2) = 25
= 84
Cr  3 8  ⋅ 54
  ⇒ 25x 2 = 1
 
Since, Tr + 1 is rational for r = 4, 12, 20, 28, 36, 44, 1
⇒ x2 =
52, 60, 68, 76, 84 25
∴ no. of rational terms = 11 1
⇒ x=±
Here, total number of terms = 85 5
∴ Number of irrational terms = 85 − 11 1
⇒ x=
= 74 5

22. (c) Given, that A − 3I 3 25. (d) Since, product of two non-null matrix can be
a null matrix.
1 1 1 1 0 0 Therefore, may be
= 1 1 1 − 3  0 1 0 A ≠ O3, B ≠ O3.
   
1 1 1  0 0 1 

Page 36 of 46
26. (a) Y x8+y8=1 29. (c) Let P(A) = 1 , P(B) = 1
2 3
1 1
P(C) = , P(D) =
4 5
X′ X Now, P(A ∪ B ∪ C ∪ D)
O
= 1 − P(A ∩ B ∩ C ∩ D)
x2+y 2=1 = 1 − P(A) P(B) P(C) P(D)
= 1 − 1 −  1 −  1 −  1 − 
1 1 1 1
Y′
 2  3  4  5
= 1 −         = 1 − =
From the graph, it is clear that there is common 1 2 3 4 1 4
region which satisfies x 2 + y 2 = 1 and x 8 + y 8 < 1  2  3  4   5 5 5
Q P∩T = P
30. (d) Given that,
27. (d) We have, σ(X) = 26
. = var(x)
x2 σ(1 − 4 x) = var(1 − 4 x)
f (x) = x −
2
1 + x2
= 16var(x)
(− x)2
∴ f (− x) = (− x)2 − =4× var(x) = 4 × 26
. = 10.4
1 + (− x)2
x2 31. (a) Given that, esin x − e − sin x − 4 = 0
= x2 − = f (x)
1 + x2
Let esin x = t
∴ f (− x) = f (x).
∴ t − t− 1 − 4 = 0
So, f (x) is many one
x2 1
Again, f (x) = x 2 − ⇒ t− −4=0
t
1 + x2
⇒ t 2 − 1 − 4t = 0
x2 + x4 − x2 x4
= = 4± 2 5
1+ x 2
1 + x2 t= = 2± 5
2
∴ Range of f (x) will be [0, ∞)
Now, esin x = 2 ± 5
But, co-domain of f (x) = R
∴ Range ≠ co-domain. So, f (x) is onto. ⇒ e sin x
= 2+ 5 and esin x = 2 − 5

28. (d) We observe the following properties: ⇒ e sin x


= 2+ 5 [e x > 0]
Reflexivity : Let a be an arbitrary element of R, Also, − 1 ≤ sin x ≤ 1
Then, a ∈ R 1
∴ ≤ esin x ≤ e
⇒ 1 + a ⋅ a = 1 + a2 > 0 [Qa 2 > 0 for all a ∈ R] e
⇒ (a , a) ∈ R1 [By def. of R1 ] ∴ esin x ≠ 2 + 5
Thus, (a , a) ∈ R1 for all a ∈ R. So, R1 is reflexive on R. Hence, there is no value of x.
Symmetry : Let (a , b) ∈ R. 32. (d) Given, that angles of a triangles are 2x, 3x and
Then, (a , b) ∈ R1 7x.
⇒ 1 + ab > 0 [Qab = b for all] Since, 2x + 3x + 7 x = 180
⇒ 1 + ba > 0 ⇒ (b , a) ∈ R1 ⇒ 12x = 180º ⇒ x = 15º
Thus, (a , b) ∈ R1 ≠ (b , a) ∈ R, for all a , b ∈ R So, angles are 30º, 45º and 105º
So, R1 is symmetric on R. a
Now, = 2R
Transitivity : we observe that 1,  ∈ R1 and
1 sin 30º
 2 a
⇒ = 2 × 10 [Q R = 10 cm]
 1 , − 1 ∈ R but (1, − 1) ∉R because 1
  1 1
2  2
1 + 1 × (− 1) = 0 >/ 0 ⇒ 2a = 20 ⇒ a = 10 cm
So, R1 is not transitive on R.

Page 37 of 46
33. (b) Let the equation of line be x + y = 1 − 2= 3(3) + c
a b
− 2= 3 3 + c
Since, the line passing through a fixed point(x1 , y1).
c = − 2− 3 3

B Hence, required equation


(a , b )
(0, b)
P ∴ y = mx + c
y= 3x − 2 − 3 3
y− 3x + 2 + 3 3 = 0

O (a, 0)
y − x 3 + 2+ 3 3 = 0
A
x1 y 35. (a) Let (α , β) be the given point, let Q(x , y) be the
∴ + 1 =1 foot of the perpendicular, and let O be the origin.
a b
The line can have any direction
Since, OAPB is a rectangle, therefore the coordinate
of P will be (a , b). ∠PQO = 90º
Hence, locus of P is Point Q lies on the circle having diameter OP.
x1 y The locus of point Q.
+ 1 =1
x y (x − 0) (x − α) + (y − 0) (y − β) = 0
⇒ x 2 − xα + y 2 − yβ = 0
34. (b) Given line,
⇒ x 2 + y2 − α x − β y = 0
3x + y = 1
⇒ y=− 3x + c 36. (b) Let coordinate of the point be (α , − α)
We know that, m = − 3 [Q y = mx + c] Since, (α , − α) lie on 2ax + 4ay + c = 0
and 7bx + 3by − d = 0
m1 − m2 ∴ 2aα − 4aα + c = 0
tanθ =
1 + m1 m2 ⇒ − 2aα + c = 0
m1 − (− 3) ⇒ α=
c
…(i)
tan 60° = [Qθ = 60º]
1 + m1 (− 3) 2a
Also, 7bα − 3bα − d = 0
m1 + 3
3= m1 [tan 60º = 3] ⇒ 4bα − d = 0
1− 3 d
⇒ α= …(ii)
m1 + 3 4b
± 3=
1 − 3m1 From Eqs. (i) and (ii),
c d
=
taking + sign taking − sign 2a 4b
m1 + 3 m1 + 3 ⇒ 2ad = 4bc
+ 3= − 3= ad 4
1 − 3m1 1 − 3m1 ⇒ =
bc 2
3(1 − 3m1 ) = m1 + 3 ⇒− 3(1 − 3m1 ) ad 2
⇒ =
= m1 + 3 bc 1
⇒ 3 − 3m1 = m1 + 3 ⇒− 3 − 3m1 = m1 + 3 ⇒ ad : bc = 2 : 1

⇒ 3m1 − m1 = 2 3 37. (a) In a circle, AB is a diameter where the


coordinate of A is (p , q) and let the coordinate of
⇒ 4m1 = 0 ⇒ 2 m1 = 2 3 B(x1 , y1).
⇒ m1 = 0 ⇒ m1 = 3 Equation of circle in diameter form is
(x − p) (x − x1) + (y − q) (y − y1) = 0
∴ given point (3, − 2)
∴ y = mx + c

Page 38 of 46
⇒ x 2 − (p + x1) x + px1 + y 2 − (y1 + q) y + qy1 = 0 39. (c) Given, equation of hyperbola is
⇒ x 2 − (p + x1) x + y 2 − (y1 + q) y + px1 + qy1 = 0 x2 y2
− = 1,
Since, this circle touches X-axis cos α sin2 α
2

∴ y=0 Here, a 2 = cos2 α and b 2 = sin2 α


⇒ x 2 − (p + x1) x + px1 + qy1 = 0 [i.e. comparing with standard
Also, the discriminant of above equation will be x 2 y2
equation 2 − 2 = 1]
equal to zero because circle touches X-axis. a b
∴ (p + x1)2 = 4(px1 + qy1) We know that, foci = (± ae , 0),
⇒ p 2 + x12 + 2px1 = 4px1 + 4qy1 where, ae = a2 + b2 = cos2 α + sin2 α = 1
⇒ x12 − 2px1 + p = 4qy1
2
⇒ foci (± 1, 0), where vertices are (± cosα , 0)
⇒ (x1 − p) = 4qy1
2 1
Eccentricity, ae = 1 or e = .
cosα
Therefore, the locus of point B is
Hence, foci remains constant with change in α.
(x − p)2 = 4qy
  40. (c) Equation of the ellipse is
38. (c) I =  ax1 + bx 2 + cx 3 , ay1 + by2 + cy3  x2 y2
 a+ b+ c a+ b+ c  2
+ 2 =1
a b
P (0, 0) B (0, b)

c b T S

I Foci are S(ae , 0), T(− ae , 0).


Q R B(0, b) is the end of the minor axis.
(1, 0) a (1/2, Ö3) STB is an equilateral triangle.
2
SB = ST
1 3 ⇒ SB2 = ST 2
a=+ = 1, b = 1, c = 1
4 4
⇒ a 2e 2 + b 2 = 4a 2e 2
Centre of the circle.
⇒ b 2 = 3a 2e 2
1 × 0 + 1 × 1 + 1 × 1 
 3 / 2
= 2 ,1 × 0 + 1 × 0 + 1 × 
⇒ a 2(1 − e 2) = 3a 2e 2
 1+1+1 1+1+1  ⇒ 1 − e 2 = 3e 2
 
⇒ 4e 2 = 1
 1 3
0+1+ 0+ 0+  1
= 2, 2  ⇒ e2 =
 3 3  4
  1
  ⇒ e=
2
3 3
  Eccentricity of the ellipse, e =
1
=  2, 2  2
3 3 
 
  41. (a) Given equation is
3 1 3 1 3x 2 − 3y 2 − 18 x + 12y + 2 = 0
= × , × 
2 3 2 3 It can be written as
(x − 3)2 (y − 2)2
=  ,
1 1  − =1
 2 2
 2 2 3  13   13 
   
 3  3

Page 39 of 46
13 c
here, a = b = =
3 (2) (− 4) − (− 2) (3)
b2 a b c
∴ e = 1+ ⇒ = =
a2 12 + 16 − 8 − 8 − 8 + 6
a b c
= 1+1 [Q a = b] ⇒ = =
28 − 16 − 2
= 2
a b c
∴ Equation of the directrix are ⇒ = =
14 − 8 − 1
13
x = 3± Hence, direction ratios are (14, − 8, − 1).
6
44. (c) The given points are A(1, 2, 3) and B(3, 4, 5)
42. (c) Given equation of ellipse is
The direction ratios of line segment AB is given by
3x 2 + 4 y 2 = 48
(x 2 − x1), (y2 − y1), (z2 − z1)
3x 2 4 y 2 x2 y2 (3 − 1), (4 − 2), (5 − 3) = (2, 2, 2) = (1, 1, 1)
⇒ + =1 ⇒ + =1 i.e.
48 48 16 12 Since, the plane bisects AB at right angles, AB is the
Here, a = 4 and b = 2 3 normal to the plane. Which is n
b2 Therefore, n = $i + $j + k$
∴ e = 1−
a2 Let c be the mid-point of AB
12 4 1 1  x1 + x 2 , y1 + y2 , z1 + z2 
= 1− = = =  
16 16 4 2  2 2 2 
1 + 3 2 + 4 3 + 5
∴ Coordinates of P are = (2, 3) = c  , , 
 2 2 2 
  b2  
Q (4 cosθ, 2 3sinθ) = (2, 3) Q p =  ae ,   = c(2, 3, 4)
  a 
Let this vector be a = 2$i + 3$j + 4k$
By comparing, we get
Hence, the required equation of vector
4 cosθ = 2 and 2 3sinθ = 3
{r − (2$i + 3$j + 4k$)} ($i + $j + k$) = 0
1 3
⇒ cosθ =
and sinθ = We know that, r = x$i + y$j + zk$
2 2
π π ⇒ {(x$i + y$j + zk$) − (2$i + 3$j + 4k$)} ($i + $j + k$) = 0
⇒ cosθ = cos and sinθ = sin
3 3
⇒ (x$i + y$j + zk$) ⋅ ($i + $j + k$)
π π
⇒ θ = and θ =
3 3 = (2$i + 3$j + 4k$) ⋅ ($i + $j + k$)
π ⇒ x + y + z = 2+ 3+ 4
∴ eccentric angle of P is .
3 ⇒x + y+ z= 9
43. (b) Direction ratios of line joining the points 45. (a) It is clear that, the limit of the interior angle
(1, 2, − 3) and (− 1, − 2, 1) is (− 1 − 1, − 2 − 2, 1 + 3) of a regular polygon of n sides as n → ∞ is π or
i.e. (− 2, − 4, 4) 180º.
Let the direction ratios of the normal to the plane is 46. (a) Given,
(a , b , c), then
h{ f (x)} = x
2a + 3b + 4c = 0 …(i)
differentiating w.r.t.x, we get
and − 2a − 4b + 4c = 0 …(ii)
h′ { f (x)}. f ′(x) = 1
By Eqs. (i) and (ii), we get 1
⇒ f ′(x) =
h′ { f (x)}
1
a b ⇒ f ′(x) =
= 1
3(4) − (4) (− 4) (− 2) (4) − (2) (4)
1 + log{ f (x)}

Page 40 of 46
Q h′(x) = 1  f (3) − f ()
1
⇒ = f ′(c)
 1 + log x  2
  f (3) − f ()
1
(given)  ⇒ = [ f | c |2 + 4]
2
⇒ f ′(x) = 1 + log( f (x))
(Q f ′(c) = | f (c) |2 + 4)
47. (c) We have, ⇒ f (3) − f ()
1 = 2⋅ | f (c) |2 + 8
f (x) = cos x 2
⇒ f (3) − f ()
1 ≥8
Let T be the period of f (x). Then It is clear from the given options that (B) and (C) the
f (x + T) = f (x) correct.
⇒ cos(x + T)2 = cos x 2
51. (c) Let f (x) = x 2 + 2x + 3
But there is no value of T for which
− D − (4 − 12) 8
cos(x + T)2 = cos x 2 a = f (x)min = = = =2
4a 4 4
∴ f (x) is not periodic 1 − cosθ 1 − 1 + 2sin2 θ/2
and b = lim = lim
48. (c) Let L = lim (e x + x)1 / x θ→ 0 θ2 θ→ 0 θ2
x → 0+
2sin θ/2
2
1  sin2 θ/2 
= lim = lim ⋅  2 
log(e x + x) θ → 0 (θ/2) ⋅ 4
 (θ/2) 
2 θ→ 0 2
⇒ log L = lim
+
x→ 0 x
1 sin2 θ/2
1 = ⋅ lim
⋅ ex + 1 2 θ → 0 (θ/2)2
(e x + x)
⇒ log L = lim 1 1
x → 0+ 1 = ⋅1 =
2 2
[using L’ Hospital rule]
1
ex + 1 b=
⇒ log L = lim x 2
x→ 0 e + x
+
n
∑a ⋅ bn − r
r
Now,
⇒ log L = 2
r=0
⇒ L = e2 n n−r
r 1 
49. (a) Let g(x) = e −x
f (x)
= ∑ (2)  
 2
r=0
∴ g′(x) = e − x f ′(x) − e − x f (x) n
∑2 ⋅2
( r − n)
−x
= r
=e ( f ′(x) − f (x)) r=0
Since, f ′(x) > f (x), so f ′(x) − f (x) > 0 n
∑2
2r − n
=
∴ e − x ( f ′(x) − f (x)) > 0
r=0
⇒ g′(x) > 0 n
= 2− n ∑2
2r
∴ g(x) is an increasing function.
r=0
Now, g(x) > g(0) ∀ x > 0
⇒ e − x f (x) > e 0 f (0) = 2− n [1 + 22 + 24 + 26 + K + 22n ]

⇒ e − x f (x) > 0 [Q f (0) = 0] 1 ⋅ (22)n + 1 − 1   a(r n − 1) 


= 2− n   Q Sn = 
 2 −1
2
  r −1 
⇒ f (x) > 0, ∀x > 0
 4n + 1 − 1 
50. (b, c) Given that f : [1, 3] → R be a continuous and = 2− n  
 3 
differentiable in (1, 3).
and f ′(x) = | f (x) |2 + 4 4n + 1 − 1
=
By applying LMVT, there exist at least one point 3⋅ 2n
c ∈(1, 3) such that 52. (a) Let a = 4, b = 1 and n = 2
f (3) − f ()
1  f (b) − f (a) 
= f ′(c) Q f ′(c) = ∴ I(n) = a1 / n − b1 / n
3−1  b − a 

Page 41 of 46
= 41 / 2 − 11 / 2 = 2 − 1 = 1
P3 (5/4, 1/2)
and J(n) = (a − b)1 / n
P4 (5/4, 3/8)
= (4 − 1)1 / 2 = 3 P2 (1, 1/2)
∴ J(n) > I(n) (P5 (21/16, 3/8))
Similarly, we can prove for other values of a , b , n. O P1 (1, 0)

53. (d) Given, 5 5 21


Here, x1 = 1, x 2 = 1, x 3 = , x 4 = and x 5 =
|α$ |= |β$ | = | γ$ | = 1 4 4 16
1 1 1 3 3
and α$ × (β$ × γ$) = (β$ + γ$) y1 = 0, y2 = , y3 = , y4 = and y5 =
2 2 2 8 8
1 $ $ 1 1 1 1 4
$ $
Now, α × (β × γ) = (β + γ)
$ ∴ xn = 1 + + 2 + K ∞ = = =
4 4 1 3 3
2 1−
1 1 4 4
$ $
⇒ (α$ ⋅ γ$)β − (α$ ⋅ β) γ$ = β$ + γ$
1 1
2 2 and yn = 0 + − + K ∞
On comparing we get, 2 8
1 1 1 1
− (α$ ⋅ β$) = 2
2 = 2 = 2 = 2=
1  1 
1 −  −  1 + 
1 5 5
$
⇒ |α ||β | cosθ = −
$  4  4 4
2
∴ (α , β) =  ,
1 4 2
⇒ cosθ = − [Q|α$ | = |β$ | = 1] 
2 3 5

⇒ θ = 120º or 56. (a) | z | + | z − 1 | a is complex number
3
| z | + | − (1 − z) | |a|= |− a|
54. (d) The position vectors of the points A, B, C and | z | + |1 − z | {Q| a | = | − a |}
D are 3$i − 2$j − k$ , 2$i − 3$j + 2k$ , 5$i − $j + 2k$ and |z| + |1 − z|≥ |z + 1 − z |
4$i − $j − λk$ respectively. {Q|z1| + |z2|≥ |z1 + z2|}
BA = (3$i − 2$j − k$) − (2$i − 3$j + 2k$) = $i + $j − 3k$ | z | + |1 − z |≥ |1 |
| z | + |1 − z |≥ 1
CA = (3$i − 2$j − k$) − (5$i − $j + 2k$) = − 2$i − $j − 3k$
| z | + |−(z − 1) |≥ 1
DA = (3$i − 2$j − k$) − (4$i − $j + λk$) | z | + | z − 1|≥ 1
= − $i − $j − (1 + λ)k$ Minimum value of| z | + | z − 1 | = 1
These are coplanar, so 57. (c) Given, system of equations
1 1 −3 λx + y + 3z = 0
− 2 −1 −3 =0 2x + µ y − z = 0
− 1 − 1 − (1 + λ) 5x + 7 y + z = 0
−1 −1 3 System has infinitely many solutions in R, if
λ 1 3
= 2 1 3 =0
2 µ −1 = 0
1 1 1+ λ
5 7 1
− 11
[ + λ − 3] + 1[2 + 2λ − 3] + 3[2 − 1] = 0
⇒ λ(µ + 7) − 1(2 + 5) + 314
( − 5 µ) = 0
⇒ − 1[λ − 2] + 1[2λ − 1] + 3 = 0
⇒ λ(µ + 7) − 7 + 3 (14 − 5 µ) = 0
⇒ − λ + 2 + 2λ − 1 + 3 = 0
⇒ λµ + 7λ − 7 + 42 − 15 µ = 0
⇒ λ+ 4=0
⇒ λ=−4

55. (b) According to the given information in ⇒ λµ + 7λ − 15 µ + 35 = 0


question, we can draw the situation of particle at By checking the options, we get option (c)
different stages as following (λ = 1, µ = 3) satisfies the given equation.

Page 42 of 46
2
58. (c) Let A = {1, 2, 3}  b2 
e2 =  1 + 2  > 2
and B = {2, 3, 4}  a 

∴ e> 2
1 3
62. (a) Given, lim + x  q 
2 4 x→ 0 p  x 
3 x  q  q 
= lim  −  
x → 0+ p  x  x
here, f − 1 (3) = 1 =
[q] x  q 
−  
−1 −1 p p x
f (4) = 2 and f (2) = 3
[q] [q]
It is clear that option (C) f − 1 (A − B) = f − 1 (A) = − 0=
p p
− f − 1 (B) is correct.
63. (b) Given,
59. (b) We have, f (x) = x 4 − 4 x 3 + 4 x 2 + c
gof : S → U is an onto function.
Q 1 =1 + c
f ()
Let Z be any arbitrary element such that Z ∈ U.
and f (2) = 24 − 4(2)3 + 4(2)2 + c
Now, gof : S → U is onto
⇒ gof (x) = Z, for x ∈ S = 16 − 32 + 16 + c
⇒ g( f (x)) = Z =c
⇒ g(y) = Z, where y = f (x) ∈ T for all Z ∈ U Now, f ()1 ⋅ f (2) = c(1 + c)
∴ For all Z ∈ U, there exists y = f (x) ∈ T such that 1 ⋅ f (2) < 0
Let f ()
g(y) = Z ⇒ c(1 + c) < 0 ⇒ c ∈ (− 1, 0)
∴ g : T → U is an onto function. ∴ For c ∈ (− 1, 0), f (x) has exactly one zero is (1, 2).

60. (a) Since, Q is the point for which PQ is bisected 64. (a)
Y
perpendicularly by the initial line (X-axis).
Therefore, Q will be the image of P in X-axis x2 – 1 cos x
∴ Coordinates of Q is (2, π / 4)

Y X′ O X
Q (2, π/4)

Y′
X′ X It is clear from the graph that it intersects at exactly
two points.

65. (c) We have,


Y′
P
) 2, –4π ) dx
= 1and y =
10
dt x
10
Now, y =
61. (b) Let the length of conjugate axis = b x
and the length of transvers axis = a On differentiating both the sides w.r.t. t, we get
Given that, b > a dy − 10 dx
⇒ = 2 ×
Now, b2 > a2 dt x dt
− 10
b2 = ×1
∴ Eccentricity (e) = 1 + (5)2
a2
− 10 − 2  x = 10, dx = 1
= =
25 5  dt 

Page 43 of 46
2  f 
2
∴ y decreases at the rate of unit per second.
5 ⇒ t 2 = m2   …(i)
h− f
66. (b,d) We have, 1
1 Again, n + x = f (t + m)2
2
In = ∫x
n
tan− 1 x dx
1 2 1
0 ⇒ h + ht = f (t + m)2 …(ii)
2 2
 n + 1 1 n+1
x 1 x 1
=  tan− 1 x ⋅  − ∫0 n + 1 ⋅ 1 + dx From Eqs. (i) and (ii)
 n + 1 0 x2 m 2(h − f)
=
π 1 1 x2 ⋅ xn − 1 n mfh
= − ∫
4(n + 1) n + 1 0 1 + x 2
dx
m2 fh
⇒ (h − f )n =
π 1 1 (1 + x 2) x n − 1 − x n − 1 2
= − ∫
4(n + 1) n + 1 0 1 + x2
dx
68. (b) We have,
π 1 1 1 1 x n −1 y = x + 1 and y = cos x
x n − 1 dx +
4(n + 1) n + 1 ∫0 ∫0 1 +
= − dx
n+1 x 2

n 1
π 1 x 1 (0, 1)
= −   +
4(n + 1) n + 1  n  0 n + 1
[x n − 1 ⋅ tan− 1 x]1 − 1 tan− 1 x ⋅ (n − 1) x n − 2 dx (π/2, 0)
 0 ∫0 (–1, 0)

π 1 π n −1
= − + −
4(n + 1) n(n + 1) 4(n + 1) n + 1
1 n−2 π /2
∫0 x tan− 1 x dx
0
∴ Require area = ∫− 1 (x + 1) dx + ∫0 cos x dx
π 1 n −1
⇒ In = − − In − 2  x2 
0
2(n + 1) n(n + 1) n + 2 = + x  + [sin x]π0 / 2
 2 − 1
n −1 π 1
⇒ In + In − 2 = −
 
n+ 2 2(n + 1) n(n + 1) = (0) −  − 1 + [1 − 0]
1
  2  
Put n = n + 2, we get
n+1 π 1 3
I n+ 2 + In = −
1 = + 1 = sq unit.
n+ 3 2(n + 3) (n + 2) (n + 3) 2 2
π 1 69. (b) We have,
⇒ (n + 3) I n + 2 + (n + 1) I n = −
2 n+ 2 x1 , x 2 be the roots of equation x 2 − 3x + a = 0
π 1 ∴ x1 + x 2 = 3 and x1 x 2 = a
∴ a n = n + 3, bn = n + 1, c n = −
2 n+ 2 Also, x 3, x 4 be the roots of equation
∴ a n and bn are in AP. x 2 − 12x + b = 0
∴ x 3 + x 4 = 12 and x 3 x 4 = b
67. (c) Let B travels x units, v = u + at
Again, x1 , x 2, x 3, x 4 are in GP
According to problem, ht = f (t + m)
∴ x1 = A, x 2 = AR, x 3 = AR2, x 4 = AR3
⇒ ht = ft + fm
Now, x1 + x 2 = 3
⇒ ht − ft = fm
⇒ A(1 + R) = 3 …(i)
⇒ t(h − f) = fm
h− f m and x 3 + x 4 = 12
⇒ = ⇒ AR2(1 + R) = 12 …(ii)
f t
 f  From Eqs. (i) and (ii), we have
⇒ t = m  R2 = 4 ⇒ R = ± 2
h− f
When, R = 2, A = 1 and when, R = − 2, A = − 3

Page 44 of 46
∴ Numbers are either 1, 2, 4, 8 or − 3, 6, − 12, 24
But x1 < x 2 < x 3 < x 4 . 3
So, x1 = 1, x 2 = 2 , x 3 = 4, x 4 = 8 2
x –y=1
∴ ab = x1 x 2 x 3 x 4 1
1/2
= 1 × 2 × 4 × 8 = 64 1 2 3 4 5 6 7 8

70. (a) Let Z = 1 − i cosθ -1 B (6 –1)


1 + 2i cosθ -2
1 + i cosθ
∴ z= -3
1 − 2i cosθ -4
Since, z is a real number, then z − z = 0 -5
1 − i cosθ 1 + i cosθ
⇒ = -6
m
1 + 2i cosθ 1 − 2i cosθ -7
A (2, –7) C
⇒ (1 − i cosθ) (1 − 2i cosθ) = (1 + i cosθ) (1 + 2i cosθ) -8
⇒ 1 − 2i cosθ − i cosθ − 2cos2 θ
Let the slope of AC be m
= 1 + 2i cosθ + i cosθ − 2cos2 θ then, AB = AC
⇒ 6 i cosθ = 0 1 1
m+ − −1
⇒ cosθ = 0 ∴ 4 = 4
π m 1
⇒ θ = (2n + 1) , n ∈ I 1− 1−
2 4 4
− 23
71. (b) We have, ⇒ m= ,1
7
 3 0 3 23
When m = − , then equation of line
A =  0 3 0 7
 
 3 0 3 − 23
y+ 7 = ( x − 2)
Again, we have 7
| A − λI | = 0 ⇒ 7 y + 49 = − 23x + 46
3− λ 0 3 ⇒ 23x + 7 y + 3 = 0
⇒ 0 3− λ 0 =0 When m = 1, then equation of line
3 0 3− λ y + 7 = ( x − 2)
⇒ x− y−9 =0
⇒ (3 − λ ) [(3 − λ ) − 0]− 0 + 3[0 − 3(3− λ)] = 0
2

73. (a, c) Given equation of hyperbola can be write as


⇒ (3 − λ )3 − 9(3 − λ ) = 0
⇒ (3 − λ ) [(3 − λ )2 − 9] = 0 x2 y2
− =1
⇒ (3 − λ ) (9 + λ − 6λ − 9) = 0
2
1 ( 5)2
⇒ (3 − λ) (λ 2 − 6 λ) = 0 here, a = 1 and b = 5
⇒ (3 − λ) λ (λ − 6) = 0 Since, y = mx ± a 2m2 − b 2
⇒ λ = 3, 0, 6
y = mx ± m2 − 5
72. (a, b) Given equations of lines are
⇒ (8 − 2m)2 = ( m2 − 5)2
x− y=7 …(i)
and x + 4y = 2 …(ii) ⇒ (8 − 2m)2 = m2 − 5
By solving Eqs. (i) and (ii), we get the point B(6, − 1) ⇒ 64 + 4m2 − 32m = m2 − 5
⇒ 3m2 − 32m + 69 = 0
22
⇒ m= ,3 [by solving equation]
3

Page 45 of 46
23 From option (c)
When m = , then equation of tangent
3 We have, g(a) = g(b) = 0
23 ⇒ g′(a) ⋅ g′(b) < 0
(y − 8) = (x − 2)
3 Again, let m(x) = g′(x) + kg(x)
⇒ 3y − 24 = 23x − 46 ⇒ m(a) = g′(a) + kg(a) = g′(a)
⇒ 23x − 3y − 46 + 24 = 0 ⇒ m(b) = g′(b) + kg(b) = g′(b)
⇒ 23x − 3y − 22 = 0 ∴ m(a) ⋅ m(b) = g′(a) ⋅ g′(b) < 0
When m = 3, then equation of tangent ∴ m(x) = 0 has root between (a , b).
(y − 8) = 3(x − 2) So, option (a) and (c) are correct.
⇒ y − 8 = 3x − 6 3
⇒ 3x − y − 6 + 8 = 0 75. (d) Given, f (x) = x − sin πx + 3
4
⇒ 3x − y + 2 = 0
differentiating w.r.t. x, we get
Hence, option (a) and (c) are correct.
3x 2
f ′(x) = − π cos(πx)
74. (a, c) From option (a) 4
We have, at x = − 2, f(− 2) = 1
f (a) = f (b) = 0 and at x = 2, f(2) = 5
⇒ f ′(a) ⋅ f ′(b) < 0
Again, let h(x) = f ′(x) + f (x) g′(x)
⇒ h(a) = f ′(a) + f (a) g′(a) = f ′(a)
and h(b) = f ′(b) + f (b) g′(b) = f ′(b) –2 2
∴ h(a) ⋅ h(b) = f ′(a) ⋅ f ′(b) < 0
∴ h(x) = 0 has root between (a, b)

Page 46 of 46

You might also like